Medsurg 2 Final

¡Supera tus tareas y exámenes ahora con Quizwiz!

A patient with MS has developed dysphagia as a result of cranial nerve dysfunction. What nursing action should the nurse consequently perform? A) Arrange for the patient to receive a low residue diet. B) Position the patient upright during feeding. C) Suction the patient following each meal. D) Withhold liquids until the patient has finished eating.

B) Position the patient upright during feeding.

A nurse assesses clients on a cardiac unit. Which clients should the nurse identify as at greatest risk for the development of acute pericarditis? (Select all that apply.) a. A 36-year-old woman with systemic lupus erythematosus (SLE) b. A 42-year-old man recovering from coronary artery bypass graft surgery c. A 59-year-old woman recovering from a hysterectomy d. An 80-year-old man with a bacterial infection of the respiratory tract e. An 88-year-old woman with a stage III sacral ulcer

A, B, D

The son of a client diagnosed with ALS asks the nurse, "Is there any chance that I could get this disease?" Which statement by the nurse would be most appropriate? 1. "It must be scary to think you might get this disease." 2. "No, this disease is not genetic or contagious." 3. "ALS does have a genetic factor and runs in families." 4. "If you are exposed to the same virus, you may get the disease."

3. "ALS does have a genetic factor and runs in families." There is a genetic factor with ALS that is linked to a chromosome 21 defect.

The nurse is admitting a client to a medical floor with a diagnosis of adenocarcinoma of the rectosigmoid colon. Which assessment data support this diagnosis? 1. The client reports up to 20 bloody stools per day. 2. The client has a feeling of fullness after a heavy meal. 3. The client has diarrhea alternating with constipation. 4. The client complains of right lower quadrant pain.

3. The client has diarrhea alternating with constipation. The most common symptom of colon cancer is a change in bowel habits, specifically diarrhea alternating with constipation.

The nurse is caring for a client who had an esophagectomy 3 days ago and was extubated yesterday. What actions may the nurse delegate to the unlicensed assistive personnel (UAP)? (Select all that apply.) a. Assisting with position changes and getting out of bed b. Keeping the head of the bed elevated to at least 30 degrees c. Reminding the client to use the spirometer every 4 hours d. Taking and recording vital signs per hospital protocol e. Titrating oxygen based on the clients oxygen saturations

A, B, D

A nurse assesses a client who is experiencing diabetic ketoacidosis (DKA). For which manifestations should the nurse monitor the client? (Select all that apply.) a. Deep and fast respirations b. Decreased urine output c. Tachycardia d. Dependent pulmonary crackles e. Orthostatic hypotension

A, C, E DKA leads to dehydration, which is manifested by tachycardia and orthostatic hypotension. Usually clients have Kussmaul respirations, which are fast and deep. Increased urinary output (polyuria) is severe.

A client has dumping syndrome. What menu selections indicate the client understands the correct diet to manage this condition? (Select all that apply.) a. Canned unsweetened apricots b. Coffee cake c. Milk shake d. Potato soup e. Steamed broccoli

A, D

A nurse assesses a client with Cushings disease. Which assessment findings should the nurse correlate with this disorder? (Select all that apply.) a. Moon face b. Weight loss c. Hypotension d. Petechiae e. Muscle atrophy

A, D, E

A nurse in a long-term care facility is caring for an 83-year-old woman who has a history of HF and peripheral arterial disease (PAD). At present the patient is unable to stand or ambulate. The nurse should implement measures to prevent what complication? A) Aoritis B) Deep vein thrombosis C) Thoracic aortic aneurysm D) Raynauds disease

B) Deep vein thrombosis

The nurse is caring for a patient with acute pericarditis. What nursing management should be instituted to minimize complications? A) The nurse keeps the patient isolated to prevent nosocomial infections. B) The nurse encourages coughing and deep breathing. C) The nurse helps the patient with activities until the pain and fever subside. D) The nurse encourages increased fluid intake until the infection resolves.

C) The nurse helps the patient with activities until the pain and fever subside.

A nurse is collaborating with the interdisciplinary team to help manage a patients recurrent headaches. What aspect of the patients health history should the nurse identify as a potential contributor to the patients headaches? A) The patient leads a sedentary lifestyle. B) The patient takes vitamin D and calcium supplements. C) The patient takes vasodilators for the treatment of angina. D) The patient has a pattern of weight loss followed by weight gain.

C) The patient takes vasodilators for the treatment of angina.

An older adult patient has been treated for a venous ulcer and a plan is in place to prevent the occurrence of future ulcers. What should the nurse include in this plan? A) Use of supplementary oxygen to aid tissue oxygenation B) Daily use of normal saline compresses on the lower limbs C) Daily administration of prophylactic antibiotics D) A high-protein diet that is rich in vitamins

D) A high-protein diet that is rich in vitamins

The student nurse learns about risk factors for gastric cancer. Which factors does this include? (Select all that apply.) a. Achlorhydria b. Chronic atrophic gastritis c. Helicobacter pylori infection d. Iron deficiency anemia e. Pernicious anemia

a. Achlorhydria b. Chronic atrophic gastritis c. Helicobacter pylori infection e. Pernicious anemia

An older client has gastric cancer and is scheduled to have a partial gastrectomy. The family does not want the client told about her diagnosis. What action by the nurse is best? a. Ask the family why they feel this way. b. Assess family concerns and fears. c. Refuse to go along with the familys wishes. d. Tell the family that such secrets cannot be kept.

b. Assess family concerns and fears.

An emergency room nurse assesses a client after a motor vehicle crash. The nurse notices a steering wheel mark across the clients chest. Which action should the nurse take? a. Ask the client where in the car he or she was sitting during the crash. b. Assess the client by gently palpating the abdomen for tenderness. c. Notify the laboratory to draw blood for blood type and crossmatch. d. Place the client on the stretcher in reverse Trendelenburg position.

b. Assess the client by gently palpating the abdomen for tenderness. The liver is often injured by a steering wheel in a motor vehicle crash.

A nurse cares for a client who is scheduled for a paracentesis. Which intervention should the nurse delegate to an unlicensed assistive personnel (UAP)? a. Have the client sign the informed consent form. b. Assist the client to void before the procedure. c. Help the client lie flat in bed on the right side. d. Get the client into a chair after the procedure.

b. Assist the client to void before the procedure. For safety, the client should void just before a paracentesis.

The nurse is working with clients who have esophageal disorders. The nurse should assess the clients for which manifestations? (Select all that apply.) a. Aphasia b. Dysphagia c. Eructation d. Halitosis e. Weight gain

b. Dysphagia c. Eructation d. Halitosis

A nurse cares for a client with advanced Alzheimers disease. The clients caregiver states, She is always wandering off. What can I do to manage this restless behavior? How should the nurse respond? a. This is a sign of fatigue. The client would benefit from a daily nap. b. Engage the client in scheduled activities throughout the day. c. It sounds like this is difficult for you. I will consult the social worker. d. The provider can prescribe a mild sedative for restlessness.

b. Engage the client in scheduled activities throughout the day.

A client with an esophageal tumor is having extreme difficulty swallowing. For what procedure does the nurse prepare this client? a. Enteral tube feeding b. Esophageal dilation c. Nissen fundoplication d. Photodynamic therapy

b. Esophageal dilation Esophageal dilation can provide immediate relief of esophageal strictures that impair swallowing.

A nurse assesses a client who has a history of migraines. Which clinical manifestation should the nurse identify as an early sign of a migraine with aura? a. Vertigo b. Lethargy c. Visual disturbances d. Numbness of the tongue

c. Visual disturbances Early warning of impending migraine with aura usually consists of visual changes, flashing lights, or diplopia.

The 85-year-old male client diagnosed with cancer of the colon asks the nurse, "Why did I get this cancer?" Which statement is the nurse's best response? 1. "Research shows a lack of fiber in the diet can cause colon cancer." 2. "It is not common to get colon cancer at your age; it is usually in young people." 3. "No one knows why anyone gets cancer, it just happens to certain people." 4. "Women usually get colon cancer more often than men but not always."

1. "Research shows a lack of fiber in the diet can cause colon cancer." A long history of low-fiber, high-fat, and high-protein diets results in a pro- longed transit time. This allows the carcinogenic agents in the waste prod- ucts to have a greater exposure to the lumen of the colon.

The client with end-stage ALS requires a gastrostomy tube feeding. Which finding would require the nurse to hold a bolus tube feeding? 1. A residual of 125 mL. 2. The abdomen is soft. 3. Three episodes of diarrhea. 4. The potassium level is 3.4 mEq/L.

1. A residual of 125 mL. A residual (aspirated gastric contents) of greater than 50 to 100 mL indicates that the tube feeding is not being digested and that the feeding should be held.

The nursing educator is presenting a case study of an adult patient who has abnormal ventricular depolarization. This pathologic change would be most evident in what component of the ECG? A) P wave B) T wave C) QRS complex D) U wave

C) QRS complex

A nurse who provides care in a walk-in clinic assesses a wide range of individuals. The nurse should identify which of the following patients as having the highest risk for chronic pancreatitis? A) A 45-year-old obese woman with a high-fat diet B) An 18-year-old man who is a weekend binge drinker C) A 39-year-old man with chronic alcoholism D) A 51-year-old woman who smokes one-and-a-half packs of cigarettes per day

C) A 39-year-old man with chronic alcoholism

A medical nurse has admitted four patients over the course of a 12-hour shift. For which patient would assessment of ankle-brachial index (ABI) be most clearly warranted? A) A patient who has peripheral edema secondary to chronic heart failure B) An older adult patient who has a diagnosis of unstable angina C) A patient with poorly controlled type 1 diabetes who is a smoker D) A patient who has community-acquired pneumonia and a history of COPD

C) A patient with poorly controlled type 1 diabetes who is a smoker

The nurse caring for a patient with Cushing syndrome is describing the dexamethasone suppression test scheduled for tomorrow. What does the nurse explain that this test will involve? A) Administration of dexamethasone orally, followed by a plasma cortisol level every hour for 3 hours B) Administration of dexamethasone IV, followed by an x-ray of the adrenal glands C) Administration of dexamethasone orally at 11 PM, and a plasma cortisol level at 8 AM the next morning D) Administration of dexamethasone intravenously, followed by a plasma cortisol level 3 hours after the drug is administered

C) Administration of dexamethasone orally at 11 PM, and a plasma cortisol level at 8 AM the next morning

A middle-aged woman has sought care from her primary care provider and undergone diagnostic testing that has resulted in a diagnosis of MS. What sign or symptom is most likely to have prompted the woman to seek care? A) Cognitive declines B) Personality changes C) Contractures D) Difficulty in coordination

D) Difficulty in coordination The primary symptoms of MS most commonly reported are fatigue, depression, weakness, numbness, difficulty in coordination, loss of balance, spasticity, and pain. Cognitive changes and contractures usually occur later in the disease.

A patient has been diagnosed with pancreatic cancer and has been admitted for care. Following initial treatment, the nurse should be aware that the patient is most likely to require which of the following? A) Inpatient rehabilitation B) Rehabilitation in the home setting C) Intensive physical therapy D) Hospice care

D) Hospice care

A client who had a partial gastrectomy has several expected nutritional problems. What actions by the nurse are best to promote better nutrition? (Select all that apply.) a. Administer vitamin B12 injections. b. Ask the provider about folic acid replacement. c. Educate the client on enteral feedings. d. Obtain consent for total parenteral nutrition. e. Provide iron supplements for the client.

a. Administer vitamin B12 injections. b. Ask the provider about folic acid replacement. e. Provide iron supplements for the client.

A client is scheduled to have a fundoplication. What statement by the client indicates a need to review preoperative teaching? a. After the operation I can eat anything I want. b. I will have to eat smaller, more frequent meals. c. I will take stool softeners for several weeks. d. This surgery may not totally control my symptoms.

a. After the operation I can eat anything I want.

A client has dumping syndrome after a partial gastrectomy. Which action by the nurse would be most helpful? a. Arrange a dietary consult. b. Increase fluid intake. c. Limit the clients foods. d. Make the client NPO.

a. Arrange a dietary consult.

A nurse assesses a client who is recovering from an open Whipple procedure. Which action should the nurse perform first? a. Assess the clients endotracheal tube with 40% FiO2. b. Insert an indwelling Foley catheter to gravity drainage. c. Place the clients nasogastric tube to low intermittent suction. d. Start lactated Ringers solution through an intravenous catheter.

a. Assess the clients endotracheal tube with 40% FiO2. ABC's

A client is in the emergency department with an esophageal trauma. The nurse palpates subcutaneous emphysema in the mediastinal area and up into the lower part of the clients neck. What action by the nurse takes priority? a. Assess the clients oxygenation. b. Facilitate a STAT chest x-ray. c. Prepare for immediate surgery. d. Start two large-bore IVs.

a. Assess the clients oxygenation.

A nurse cares for a client who has obstructive jaundice. The client asks, Why is my skin so itchy? How should the nurse respond? a. Bile salts accumulate in the skin and cause the itching. b. Toxins released from an inflamed gallbladder lead to itching. c. Itching is caused by the release of calcium into the skin. d. Itching is caused by a hypersensitivity reaction.

a. Bile salts accumulate in the skin and cause the itching. In obstructive jaundice, the normal flow of bile into the duodenum is blocked, allowing excess bile salts to accumulate on the skin. This leads to itching, or pruritus.

A nurse cares for a client who is prescribed lactulose (Heptalac). The client states, I do not want to take this medication because it causes diarrhea. How should the nurse respond? a. Diarrhea is expected; thats how your body gets rid of ammonia. b. You may take Kaopectate liquid daily for loose stools. c. Do not take any more of the medication until your stools firm up. d. We will need to send a stool specimen to the laboratory.

a. Diarrhea is expected; thats how your body gets rid of ammonia. The purpose of administering lactulose to this client is to help ammonia leave the circulatory system through the colon.

A nurse prepares to discharge a client with chronic pancreatitis. Which question should the nurse ask to ensure safety upon discharge? a. Do you have a one- or two-story home? b. Can you check your own pulse rate? c. Do you have any alcohol in your home? d. Can you prepare your own meals?

a. Do you have a one- or two-story home? A client recovering from chronic pancreatitis should be limited to one floor until strength and activity increase.

A nurse assesses a client who is recovering from a Whipple procedure. Which assessment finding alerts the nurse to urgently contact the health care provider? a. Drainage from a fistula b. Absent bowel sounds c. Pain at the incision site d. Nasogastric (NG) tube drainage

a. Drainage from a fistula Complications of a Whipple procedure include secretions that drain from a fistula and peritonitis. Absent bowel sounds, pain at the incision site, and NG tube drainage are normal postoperative findings.

A nurse cares for a client with end-stage pancreatic cancer. The client asks, Why is this happening to me? How should the nurse respond? a. I dont know. I wish I had an answer for you, but I dont. b. Its important to keep a positive attitude for your family right now. c. Scientists have not determined why cancer develops in certain people. d. I think that this is a trial so you can become a better person because of it.

a. I dont know. I wish I had an answer for you, but I dont.

A nurse witnesses a client with late-stage Alzheimers disease eat breakfast. Afterward the client states, I am hungry and want breakfast. How should the nurse respond? a. I see you are still hungry. I will get you some toast. b. You ate your breakfast 30 minutes ago. c. It appears you are confused this morning. d. Your family will be here soon. Lets get you dressed.

a. I see you are still hungry. I will get you some toast.

An emergency department nurse assesses a client with ketoacidosis. Which clinical manifestation should the nurse correlate with this condition? a. Increased rate and depth of respiration b. Extremity tremors followed by seizure activity c. Oral temperature of 102 F (38.9 C) d. Severe orthostatic hypotension

a. Increased rate and depth of respiration Ketoacidosis decreases the pH of the blood, stimulating the respiratory control areas of the brain to buffer the effects of increasing acidosis. The rate and depth of respiration are increased (Kussmaul respirations) in an attempt to excrete more acids by exhalation.

An emergency nurse cares for a client who is experiencing an acute adrenal crisis. Which action should the nurse take first? a. Obtain intravenous access. b. Administer hydrocortisone succinate (Solu-Cortef). c. Assess blood glucose. d. Administer insulin and dextrose.

a. Obtain intravenous access. All actions are right but priority is to establish IV access.

A nurse assesses a client who is recovering from a paracentesis 1 hour ago. Which assessment finding requires action by the nurse? a. Urine output via indwelling urinary catheter is 20 mL/hr b. Blood pressure increases from 110/58 to 120/62 mm Hg c. Respiratory rate decreases from 18 to 14 breaths/min d. A decrease in the clients weight by 6 kg

a. Urine output via indwelling urinary catheter is 20 mL/hr Rapid removal of ascetic fluid causes decreased abdominal pressure, which can contribute to hypovolemia.

A nurse teaches a client with hepatitis C who is prescribed ribavirin (Copegus). Which statement should the nurse include in this clients discharge education? a. Use a pill organizer to ensure you take this medication as prescribed. b. Transient muscle aching is a common side effect of this medication. c. Follow up with your provider in 1 week to test your blood for toxicity. d. Take your radial pulse for 1 minute prior to taking this medication.

a. Use a pill organizer to ensure you take this medication as prescribed. Treatment of hepatitis C with ribavirin takes up to 48 weeks, making compliance a serious issue.

A nurse assesses a client with pericarditis. Which assessment finding should the nurse expect to find? a. Heart rate that speeds up and slows down b. Friction rub at the left lower sternal border c. Presence of a regular gallop rhythm d. Coarse crackles in bilateral lung bases

b. Friction rub at the left lower sternal border

After teaching a client who has been diagnosed with hepatitis A, the nurse assesses the clients understanding. Which statement by the client indicates a correct understanding of the teaching? a. Some medications have been known to cause hepatitis A. b. I may have been exposed when we ate shrimp last weekend. c. I was infected with hepatitis A through a recent blood transfusion. d. My infection with Epstein-Barr virus can co-infect me with hepatitis A.

b. I may have been exposed when we ate shrimp last weekend. The route of acquisition of hepatitis A infection is through close personal contact or ingestion of contaminated water or shellfish.

After teaching a client who has alcohol-induced cirrhosis, a nurse assesses the clients understanding. Which statement made by the client indicates a need for additional teaching? a. I cannot drink any alcohol at all anymore. b. I need to avoid protein in my diet. c. I should not take over-the-counter medications. d. I should eat small, frequent, balanced meals.

b. I need to avoid protein in my diet. Based on the degree of liver involvement and decreased function, protein intake may have to be decreased. However, some protein is necessary for the synthesis of albumin and normal healing.

A nurse obtains a clients health history at a community health clinic. Which statement alerts the nurse to provide health teaching to this client? a. I drink two glasses of red wine each week. b. I take a lot of Tylenol for my arthritis pain. c. I have a cousin who died of liver cancer. d. I got a hepatitis vaccine before traveling.

b. I take a lot of Tylenol for my arthritis pain. Acetaminophen (Tylenol) can cause liver damage if taken in large amounts.

A client has a recurrence of gastric cancer and is in the gastrointestinal clinic crying. What response by the nurse is most appropriate? a. Do you have family or friends for support? b. Id like to know what you are feeling now. c. Well, we knew this would probably happen. d. Would you like me to refer you to hospice?

b. Id like to know what you are feeling now.

A nurse assesses a client with Huntington disease. Which motor changes should the nurse monitor for in this client? a. Shuffling gait b. Jerky hand movements c. Continuous chewing motions d. Tremors of the hands

b. Jerky hand movements An imbalance between excitatory and inhibitory neurotransmitters leads to uninhibited motor movements, such as brisk, jerky, purposeless movements of the hands, face, tongue, and legs.

A nurse cares for a client with hepatic portal-systemic encephalopathy (PSE). The client is thin and cachectic in appearance, and the family expresses distress that the client is receiving little dietary protein. How should the nurse respond? a. A low-protein diet will help the liver rest and will restore liver function. b. Less protein in the diet will help prevent confusion associated with liver failure. c. Increasing dietary protein will help the client gain weight and muscle mass. d. Low dietary protein is needed to prevent fluid from leaking into the abdomen.

b. Less protein in the diet will help prevent confusion associated with liver failure. A low-protein diet is ordered when serum ammonia levels increase and/or the client shows signs of PSE. A low-protein diet helps reduce excessive breakdown of protein into ammonia by intestinal bacteria. Encephalopathy is caused by excess ammonia.

A client with an esophageal tumor has difficulty swallowing and has been working with a speech-language pathologist. What assessment finding by the nurse indicates that the priority goal for this problem is being met? a. Choosing foods that are easy to swallow b. Lungs clear after meals and snacks c. Properly performing swallowing exercises d. Weight unchanged after 2 weeks

b. Lungs clear after meals and snacks Client at risk for aspiration

A nurse plans care for a client with acute pancreatitis. Which intervention should the nurse include in this clients plan of care to reduce discomfort? a. Administer morphine sulfate intravenously every 4 hours as needed. b. Maintain nothing by mouth (NPO) and administer intravenous fluids. c. Provide small, frequent feedings with no concentrated sweets. d. Place the client in semi-Fowlers position with the head of bed elevated.

b. Maintain nothing by mouth (NPO) and administer intravenous fluids. The client should be kept NPO to reduce GI activity and reduce pancreatic enzyme production.

A nurse cares for a client who is recovering from an open Whipple procedure. Which action should the nurse take? a. Clamp the nasogastric tube. b. Place the client in semi-Fowlers position. c. Assess vital signs once every shift. d. Provide oral rehydration.

b. Place the client in semi-Fowlers position. Semi-Fowlers position to reduce tension on the suture line and anastomosis sites, setting the nasogastric tube to low suction to remove free air buildup and pressure, assessing vital signs frequently to assess fluid and electrolyte complications, and providing intravenous fluids.

A nurse cares for a client who has cirrhosis of the liver. Which action should the nurse take to decrease the presence of ascites? a. Monitor intake and output. b. Provide a low-sodium diet. c. Increase oral fluid intake. d. Weigh the client daily.

b. Provide a low-sodium diet.

A client has a nasogastric (NG) tube after a Nissen fundoplication. The nurse answers the call light and finds the client vomiting bright red blood with the NG tube lying on the floor. What action should the nurse take first? a. Notify the surgeon. b. Put on a pair of gloves. c. Reinsert the NG tube. d. Take a set of vital signs.

b. Put on a pair of gloves.

A nurse is teaching a client who experiences migraine headaches and is prescribed a beta blocker. Which statement should the nurse include in this clients teaching? a. Take this drug only when you have prodromal symptoms indicating the onset of a migraine headache. b. Take this drug as ordered, even when feeling well, to prevent vascular changes associated with migraine headaches. c. This drug will relieve the pain during the aura phase soon after a headache has started. d. This medication will have no effect on your heart rate or blood pressure because you are taking it for migraines

b. Take this drug as ordered, even when feeling well, to prevent vascular changes associated with migraine headaches. Beta blockers are prescribed as prophylactic treatment to prevent the vascular changes that initiate migraine headaches.

A nurse cares for a client with hepatitis C. The clients brother states, I do not want to contract this infection, so I will not go into his hospital room. How should the nurse respond? a. If you wear a gown and gloves, you will not get this virus. b. Viral hepatitis is not spread through casual contact. c. This virus is only transmitted through a fecal specimen. d. I can give you an update on your brothers status from here.

b. Viral hepatitis is not spread through casual contact. Viral hepatitis, or hepatitis C, is spread via blood-to-blood transmission and is associated with illicit IV drug needle sharing, blood and organ transplantation, accidental needle sticks, unsanitary tattoo equipment, and sharing of intranasal cocaine paraphernalia.

A nurse evaluates prescriptions for a client with chronic atrial fibrillation. Which medication should the nurse expect to find on this clients medication administration record to prevent a common complication of this condition? a. Sotalol (Betapace) b. Warfarin (Coumadin) c. Atropine (Sal-Tropine) d. Lidocaine (Xylocaine)

b. Warfarin (Coumadin) Atrial fibrillation puts clients at risk for developing emboli.

A telehealth nurse speaks with a client who is recovering from a liver transplant 2 weeks ago. The client states, I am experiencing right flank pain and have a temperature of 101 F. How should the nurse respond? a. The anti-rejection drugs you are taking make you susceptible to infection. b. You should go to the hospital immediately to have your new liver checked out. c. You should take an additional dose of cyclosporine today. d. Take acetaminophen (Tylenol) every 4 hours until you feel better.

b. You should go to the hospital immediately to have your new liver checked out. Fever, right quadrant or flank pain, and jaundice are signs of liver transplant rejection

A nurse works on the surgical unit. After receiving the hand-off report, which client should the nurse see first? a. Client who underwent diverticula removal with a pulse of 106/min b. Client who had esophageal dilation and is attempting first postprocedure oral intake c. Client who had an esophagectomy with a respiratory rate of 32/min d. Client who underwent hernia repair, reporting incisional pain of 7/10

c. Client who had an esophagectomy with a respiratory rate of 32/min This indicates early signs of sepsis

A nurse assesses a client with diabetes mellitus 3 hours after a surgical procedure and notes the clients breath has a fruity odor. Which action should the nurse take? a. Encourage the client to use an incentive spirometer. b. Increase the clients intravenous fluid flow rate. c. Consult the provider to test for ketoacidosis. d. Perform meticulous pulmonary hygiene care.

c. Consult the provider to test for ketoacidosis.

A nurse assesses a client with early-onset multiple sclerosis (MS). Which clinical manifestation should the nurse expect to find? a. Hyperresponsive reflexes b. Excessive somnolence c. Nystagmus d. Heat intolerance

c. Nystagmus Early signs and symptoms of MS include changes in motor skills, vision, and sensation

A client has gastroesophageal reflux disease (GERD). The provider prescribes a proton pump inhibitor. About what medication should the nurse anticipate teaching the client? a. Famotidine (Pepcid) b. Magnesium hydroxide (Maalox) c. Omeprazole (Prilosec) d. Ranitidine (Zantac)

c. Omeprazole (Prilosec) Famotidine and ranitidine are histamine blockers. Maalox is an antacid.

A client has a nasogastric (NG) tube. What action by the nursing student requires the registered nurse to intervene? a. Checking tube placement every 4 to 8 hours b. Monitoring and documenting drainage from the NG tube c. Pinning the tube to the gown so the client cannot turn the head d. Providing oral care every 4 to 8 hours

c. Pinning the tube to the gown so the client cannot turn the head

A nurse delegates care for a client with early-stage Alzheimers disease to an unlicensed assistive personnel (UAP). Which statement should the nurse include when delegating this clients care? a. If she is confused, play along and pretend that everything is okay. b. Remove the clock from her room so that she doesnt get confused. c. Reorient the client to the day, time, and environment with each contact. d. Use validation therapy to recognize and acknowledge the clients concerns.

c. Reorient the client to the day, time, and environment with each contact.

After teaching a client who is prescribed pancreatic enzyme replacement therapy, the nurse assesses the clients understanding. Which statement made by the client indicates a need for additional teaching? a. The capsules can be opened and the powder sprinkled on applesauce if needed. b. I will wipe my lips carefully after I drink the enzyme preparation. c. The best time to take the enzymes is immediately after I have a meal or a snack. d. I will not mix the enzyme powder with food or liquids that contain protein.

c. The best time to take the enzymes is immediately after I have a meal or a snack. The enzymes should be taken immediately before eating meals or snacks.

A client is scheduled for a total gastrectomy for gastric cancer. What preoperative laboratory result should the nurse report to the surgeon immediately? a. Albumin: 2.1 g/dL b. Hematocrit: 28% c. Hemoglobin: 8.1 mg/dL d. International normalized ratio (INR): 4.2

d. International normalized ratio (INR): 4.2 An INR as high as 4.2 poses a serious risk of bleeding during the operation and should be reported.

The client is diagnosed with cancer of the head of the pancreas. Which signs and symptoms should the nurse expect to assess? 1. Clay-colored stools and dark urine. 2. Night sweats and fever. 3. Left lower abdominal cramps and tenesmus. 4. Nausea and coffee-ground emesis.

1. Clay-colored stools and dark urine. The client will have jaundice, clay- colored stools, and tea-colored urine resulting from blockage of the bile drainage.

The nurse is caring for a patient with Addisons disease who is scheduled for discharge. When teaching the patient about hormone replacement therapy, the nurse should address what topic? A) The possibility of precipitous weight gain B) The need for lifelong steroid replacement C) The need to match the daily steroid dose to immediate symptoms D) The importance of monitoring liver function

B) The need for lifelong steroid replacement

The nurse is caring for an adult client diagnosed with GERD. Which condition is the most common comorbid disease associated with GERD? 1. Adult-onset asthma. 2. Pancreatitis. 3. Peptic ulcer disease. 4. Increased gastric emptying.

1. Adult-onset asthma. Of adult-onset asthma cases, 80% to 90% are caused by gastroesophageal reflux disease (GERD).

The client is diagnosed with ALS. Which client problem would be most appropriate for this client? 1. Disuse syndrome. 2. Altered body image. 3. Fluid and electrolyte imbalance. 4. Alteration in pain.

1. Disuse syndrome. Disuse syndrome is associated with complications of bedrest. Clients with ALS cannot move and reposition themselves, and they frequently have altered nutritional and hydration status.

The public health nurse is discussing hepatitis B with a group in the community. Which health promotion activities should the nurse discuss with the group? Select all that apply. 1. Do not share needles or equipment. 2. Use barrier protection during sex. 3. Get the hepatitis B vaccine. 4. Obtain immune globulin injections. 5. Avoid any type of hepatotoxic medications.

1. Do not share needles or equipment. 2. Use barrier protection during sex. 3. Get the hepatitis B vaccine. 1. Hepatitis B can be transmitted by sharing any type of needles, especially those used by drug abusers. 2. Hepatitis B can be transmitted through sexual activity; therefore, the nurse should recommend abstinence, mutual monogamy, or barrier protection. 3. Three doses of hepatitis B vaccine pro- vide immunity in 90% of healthy adults. 4. Immune globulin injections are adminis- tered as postexposure prophylaxis (after being exposed to hepatitis B), but encour- aging these injections is not a health promotion activity. 5. Hepatotoxic medications should be avoided in clients who have hepatitis or who have had hepatitis. The health-care provider prescribes medications, and the layperson does not know which medica- tions are hepatotoxic.

The nurse writes the client problem of "altered sexual functioning" for a male client diagnosed with multiple sclerosis (MS). Which intervention should be implemented? 1. Encourage the couple to explore alternative ways of maintaining intimacy. 2. Make an appointment with a psychotherapist to counsel the couple. 3. Explain daily exercise will help increase libido and sexual arousal. 4. Discuss the importance of keeping physically calm during sexual intercourse.

1. Encourage the couple to explore alternative ways of maintaining intimacy. This will assist the client and signifi- cant other to maintain a close relation- ship without putting undue pressure on the client.

The male client diagnosed with multiple sclerosis states he has been investigating alternative therapies to treat his disease. Which intervention is most appropriate by the nurse? 1. Encourage the therapy if it is not contraindicated by the medical regimen. 2. Tell the client only the health-care provider should discuss this with him. 3. Ask how his significant other feels about this deviation from the medical regimen. 4. Suggest the client research an investigational therapy instead.

1. Encourage the therapy if it is not contraindicated by the medical regimen. The nurse should listen without being judgmental about any alterative ther- apy the client is considering. Alterna- tive therapies, such as massage and relaxation, are frequently beneficial and enhance the medical regimen.

The client newly diagnosed with multiple sclerosis (MS) states, "I don't understand how I got multiple sclerosis. Is it genetic?" On which statement should the nurse base the response? 1. Genetics may play a role in susceptibility to MS, but the disease may be caused by a virus. 2. There is no evidence suggesting there is any chromosomal involvement in developing MS. 3. Multiple sclerosis is caused by a recessive gene, so both parents had to have the gene for the client to get MS. 4. Multiple sclerosis is caused by an autosomal dominant gene on the Y chromosome, so only fathers can pass it on.

1. Genetics may play a role in susceptibility to MS, but the disease may be caused by a virus. The exact cause of MS is not known, but there is a theory stating a slow virus is partially responsible. A failure of a part of the immune system may also be at fault. A genetic predisposition involv- ing chromosomes 2, 3, 7, 11, 17, 19, and X may be involved.

The public health nurse is teaching day-care workers. Which type of hepatitis is transmitted by the fecal-oral route via contaminated food, water, or direct contact with an infected person? 1. Hepatitis A. 2. Hepatitis B. 3. Hepatitis C. 4. Hepatitis D.

1. Hepatitis A. The hepatitis A virus is in the stool of infected people and takes up to two (2) weeks before symptoms develop.

The nurse is administering a pancreatic enzyme to the client diagnosed with chronic pancreatitis. Which statement best explains the rationale for administering this medication? 1. It is an exogenous source of protease, amylase, and lipase. 2. This enzyme increases the number of bowel movements. 3. This medication breaks down in the stomach to help with digestion. 4. Pancreatic enzymes help break down fat in the small intestine.

1. It is an exogenous source of protease, amylase, and lipase. Pancreatic enzymes enhance the digestion of starches (carbohydrates) in the gastrointestinal tract by supply- ing an exogenous (outside) source of the pancreatic enzymes protease, amylase, and lipase.

The nurse caring for a client one (1) day postoperative sigmoid resection notes a moderate amount of dark reddish brown drainage on the midline abdominal incision. Which intervention should the nurse implement first? 1. Mark the drainage on the dressing with the time and date. 2. Change the dressing immediately using sterile technique. 3. Notify the health-care provider immediately. 4. Reinforce the dressing with a sterile gauze pad.

1. Mark the drainage on the dressing with the time and date. The nurse should mark the drainage on the dressing to determine if active bleeding is occurring, because dark reddish-brown drainage indicates old blood. This allows the nurse to assess what is actually happening.

The client with an acute exacerbation of chronic pancreatitis has a nasogastric tube. Which interventions should the nurse implement? Select all that apply. 1. Monitor the client's bowel sounds. 2. Monitor the client's food intake. 3. Assess the client's intravenous site. 4. Provide oral and nasal care. 5. Monitor the client's blood glucose.

1. Monitor the client's bowel sounds. 3. Assess the client's intravenous site. 4. Provide oral and nasal care. 5. Monitor the client's blood glucose. 1. The return of bowel sounds indicates the return of peristalsis, and the naso- gastric suction is usually discontinued within 24 to 48 hours thereafter. 2. The client will be NPO secondary to the chronic pancreatitis, and the client cannot eat with a nasogastric tube. 3. The nurse should assess for signs of infection or infiltration. 4. Fasting and the N/G tube increase the client's risk for mucous membrane irritation and breakdown. 5. Blood glucose levels are monitored because clients with chronic pancreati- tis can develop diabetes mellitus.

The nurse is admitting a client diagnosed with multiple sclerosis. Which clinical manifestation should the nurse assess? Select all that apply. 1. Muscle flaccidity. 2. Lethargy. 3. Dysmetria. 4. Fatigue. 5. Dysphagia.

1. Muscle flaccidity. 3. Dysmetria. 4. Fatigue. 5. Dysphagia. 1. Muscle flaccidity is a hallmark symptom of MS. 2. Lethargy is the state of prolonged sleepiness or serious drowsiness and is not associated with MS. 3. Dysmetria is the inability to control muscular action characterized by overestimating or underestimating range of movement. 4. Fatigue is a symptom of MS. 5. Dysphagia, or difficulty swallowing, is associated with MS.

The nurse is preparing to administer a.m. medications to clients. Which medication should the nurse question before administering? 1. Pancreatic enzymes to the client who has finished breakfast. 2. The pain medication, morphine, to the client who has a respiratory rate of 20. 3. The loop diuretic to the client who has a serum potassium level of 3.9 mEq/L. 4. The beta blocker to the client who has an apical pulse of 68 bpm.

1. Pancreatic enzymes to the client who has finished breakfast. Pancreatic enzymes must be adminis- tered with meals to enhance the digestion of starches and fats in the gastrointestinal tract.

The client is in the terminal stage of ALS. Which intervention should the nurse implement? 1. Perform passive ROM every two (2) hours. 2. Maintain a negative nitrogen balance. 3. Encourage a low-protein, soft-mechanical diet. 4. Turn the client and have him cough and deep breathe every shift.

1. Perform passive ROM every two (2) hours. Contractures can develop within a week because extensor muscles are weaker than flexor muscles. If the client cannot perform ROM exercises, then the nurse must do it for him—passive ROM.

The nurse writes the problem "imbalanced nutrition: less than body requirements" for the client diagnosed with hepatitis. Which intervention should the nurse include in the plan of care? 1. Provide a high-calorie intake diet. 2. Discuss total parenteral nutrition (TPN). 3. Instruct the client to decrease salt intake. 4. Encourage the client to increase water intake.

1. Provide a high-calorie intake diet. 1. Sufficient energy is required for heal- ing. Adequate carbohydrate intake can spare protein. The client should eat approximately 16 carbohydrate kilocalories for each kilogram of ideal body weight daily.

The nurse is planning the care of a client who has had an abdominal-perineal resection for cancer of the colon. Which interventions should the nurse implement? Select all that apply. 1. Provide meticulous skin care to stoma. 2. Assess the flank incision. 3. Maintain the indwelling catheter. 4. Irrigate the JP drains every shift. 5. Position the client semirecumbent.

1. Provide meticulous skin care to stoma. 3. Maintain the indwelling catheter. 5. Position the client semirecumbent. 1.Colostomy stomas are openings through the abdominal wall into the colon, through which feces exit the body. Feces can be irritating to the abdominal skin, so careful and thorough skin care is needed. 2. There are midline and perineal incisions, not flank incisions. 3. Because of the perineal wound, the client will have an indwelling catheter to keep urine out of the incision. 4. Jackson Pratt drains are emptied every shift, but they are not irrigated. 5. The client should not sit upright because this causes pressure on the perineum.

The nurse is performing an admission assessment on a client diagnosed with GERD. Which signs and symptoms would indicate GERD? 1. Pyrosis, water brash, and flatulence. 2. Weight loss, dysarthria, and diarrhea. 3. Decreased abdominal fat, proteinuria, and constipation. 4. Midepigastric pain, positive H. pylori test, and melena.

1. Pyrosis, water brash, and flatulence. Pyrosis is heartburn, water brash is the feeling of saliva secretion as a result of reflux, and flatulence is gas—all symptoms of GERD.

The nurse is assessing a client with complaints of vague upper abdominal pain worse at night but relieved by sitting up and leaning forward. Which assessment question should the nurse ask next? 1. "Have you noticed a yellow haze when you look at things?" 2. "Does the pain get worse when you eat a meal or snack?" 3. "Have you had your amylase and lipase checked recently?" 4. "How much weight have you gained since you saw an HCP?"

2. "Does the pain get worse when you eat a meal or snack?" 2. The abdominal pain is often made worse by eating and lying supine in clients diagnosed with cancer of the pancreas.

The client diagnosed with cancer of the pancreas is being discharged to start chemotherapy in the HCP's office. Which statement made by the client indicates the client understands the discharge instructions? 1. "I will have to see the HCP every day for six (6) weeks for my treatments." 2. "I should write down all my questions so I can ask them when I see the HCP." 3. "I am sure this is not going to be a serious problem for me to deal with." 4. "The nurse will give me an injection in my leg and I will get to go home."

2. "I should write down all my questions so I can ask them when I see the HCP." The most important person in the treatment of the cancer is the client. Research has proved the more involved a client becomes in his or her care, the better the prognosis. Clients should have a chance to ask questions.

The client diagnosed with acute pancreatitis is being discharged home. Which statement by the client indicates the teaching has been effective? 1. "I should decrease my intake of coffee, tea, and cola." 2. "I will eat a low-fat diet and avoid spicy foods." 3. "I will check my amylase and lipase levels daily." 4. "I will return to work tomorrow but take it easy."

2. "I will eat a low-fat diet and avoid spicy foods." High-fat and spicy foods stimulate gas- tric and pancreatic secretions and may precipitate an acute pancreatic attack.

The client diagnosed with ALS asks the nurse, "I know this disease is going to kill me. What will happen to me in the end?" Which statement by the nurse would be most appropriate? 1. "You are afraid of how you will die?" 2. "Most people with ALS die of respiratory failure." 3. "Don't talk like that. You have to stay positive." 4. "ALS is not a killer. You can live a long life."

2. "Most people with ALS die of respiratory failure." About 50% of clients die within two (2) to five (5) years from respiratory failure, aspiration pneumonia, or another infectious process.

The client with hepatitis asks the nurse, "I went to an herbalist, who recommended I take milk thistle. What do you think about the herb?" Which statement is the nurse's best response? 1. "You are concerned about taking an herb." 2. "The herb has been used to treat liver disease." 3. "I would not take anything that is not prescribed." 4. "Why would you want to take any herbs?"

2. "The herb has been used to treat liver disease." 2. Milk thistle has an active ingredient, silymarin, which has been used to treat liver disease for more than 2,000 years. It is a powerful oxidant and promotes liver cell growth.

The male client tells the nurse he has been experiencing "heartburn" at night that awakens him. Which assessment question should the nurse ask? 1. "How much weight have you gained recently?" 2. "What have you done to alleviate the heartburn?" 3. "Do you consume many milk and dairy products?" 4. "Have you been around anyone with a stomach virus?"

2. "What have you done to alleviate the heartburn?" Most clients with GERD have been self- medicating with over-the-counter med- ications prior to seeking advice from a health-care provider. It is important to know what the client has been using to treat the problem.

The nurse enters the room of a client diagnosed with acute exacerbation of multiple sclerosis and finds the client crying. Which statement is the most therapeutic response for the nurse to make? 1. "Why are you crying? The medication will help the disease." 2. "You seem upset. I will sit down and we can talk for awhile." 3. "Multiple sclerosis is a disease that has good times and bad times." 4. "I will have the chaplain come and stay with you for a while."

2. "You seem upset. I will sit down and we can talk for awhile." This is stating a fact and offering self. Both are therapeutic techniques for conversations.

The client with ALS is admitted to the medical unit with shortness of breath, dyspnea, and respiratory complications. Which intervention should the nurse implement first? 1. Elevate the head of the bed 30 degrees. 2. Administer oxygen via nasal cannula. 3. Assess the client's lung sounds. 4. Obtain a pulse oximeter reading.

2. Administer oxygen via nasal cannula. Oxygen should be given immediately to help alleviate the difficulty breathing. Remember that oxygenation is priority.

Which client problem has priority for the client diagnosed with acute pancreatitis? 1. Risk for fluid volume deficit. 2. Alteration in comfort. 3. Imbalanced nutrition: less than body requirements. 4. Knowledge deficit.

2. Alteration in comfort. Autodigestion of the pancreas results in severe epigastric pain, accompanied by nausea, vomiting, abdominal tenderness, and muscle guarding.

The home health nurse is admitting a client diagnosed with cancer of the pancreas. Which information is the most important for the nurse to discuss with the client? 1. Determine the client's food preferences. 2. Ask the client if there is an advance directive. 3. Find out about insurance/Medicare reimbursement. 4. Explain the client should eat as much as possible.

2. Ask the client if there is an advance directive. Cancer of the pancreas has a poor prognosis; the nurse should determine if the client has executed an advance directive regarding his or her wishes.

The female nurse sticks herself with a contaminated needle. Which action should the nurse implement first? 1. Notify the infection control nurse. 2. Cleanse the area with soap and water. 3. Request postexposure prophylaxis. 4. Check the hepatitis status of the client.

2. Cleanse the area with soap and water. 2. The nurse should first clean the needle stick with soap and water and attempt stick bleed to help remove any virus injected into the skin.

The occupational health nurse is preparing a presentation to a group of factory workers about preventing colon cancer. Which information should be included in the presentation? 1. Wear a high-filtration mask when around chemicals. 2. Eat several servings of cruciferous vegetables daily. 3. Take a multiple vitamin every day. 4. Do not engage in high-risk sexual behaviors.

2. Eat several servings of cruciferous vegetables daily. Cruciferous vegetables, such as broc- coli, cauliflower, and cabbage, are high in fiber. One of the risks for cancer of the colon is a high-fat, low-fiber, and high-protein diet. The longer the tran- sit time (the time from ingestion of the food to the elimination of the waste products), the greater the chance of developing cancer of the colon.

Which instruction should the nurse discuss with the client who is in the icteric phase of hepatitis C? 1. Decrease alcohol intake. 2. Encourage rest periods. 3. Eat a large evening meal. 4. Drink diet drinks and juices.

2. Encourage rest periods. Adequate rest is needed for maintaining optimal immune function.

The client diagnosed with cancer of the head of the pancreas is two (2) days post-pancreatoduodenectomy (Whipple's procedure). Which nursing problem has the highest priority? 1. Anticipatory grieving. 2. Fluid volume imbalance. 3. Alteration in comfort. 4. Altered nutrition.

2. Fluid volume imbalance. This is a major abdominal surgery, and massive fluid volume shifts occur when this type of trauma is experienced by the body. Maintaining the circulatory system without overloading it requires extremely close monitoring.

Which diagnostic test is used to confirm the diagnosis of ALS? 1. Electromyogram (EMG). 2. Muscle biopsy. 3. Serum creatine kinase (CK). 4. Pulmonary function test.

2. Muscle biopsy. Biopsy confirms changes consistent with atrophy and loss of muscle fiber, both characteristic of ALS.

Which type of precaution should the nurse implement to protect from being exposed to any of the hepatitis viruses? 1. Airborne Precautions. 2. Standard Precautions. 3. Droplet Precautions. 4. Exposure Precautions.

2. Standard Precautions. Standard Precautions apply to blood, all body fluids, secretions, and excre- tions, except sweat, regardless of whether they contain visible blood.

The client diagnosed with liver problems asks the nurse, "Why are my stools clay-colored?" On which scientific rationale should the nurse base the response? 1. There is an increase in serum ammonia level. 2. The liver is unable to excrete bilirubin. 3. The liver is unable to metabolize fatty foods. 4. A damaged liver cannot detoxify vitamins.

2. The liver is unable to excrete bilirubin. 2. Bilirubin, the by-product of red blood cell destruction, is metabolized in the liver and excreted via the feces, which causes the feces to be brown in color. If the liver is damaged, the bilirubin is excreted via the urine and skin.

The client diagnosed with multiple sclerosis is scheduled for a magnetic resonance imaging (MRI) scan of the head. Which information should the nurse teach the client about the test? 1. The client will have wires attached to the scalp and lights will flash off and on. 2. The machine will be loud and the client must not move the head during the test. 3. The client will drink a contrast medium 30 minutes to one (1) hour before the test. 4. The test will be repeated at intervals during a five (5)- to six (6)-hour period.

2. The machine will be loud and the client must not move the head during the test. MRI scans require the client to lie still and not move the body; the client should be warned about the loud noise.

The client had a total pancreatectomy and splenectomy for cancer of the body of the pancreas. Which discharge instructions should the nurse teach? Select all that apply. 1. Keep a careful record of intake and output. 2. Use a stool softener or bulk laxative regularly. 3. Use correct insulin injection technique. 4. Take the pain medication before the pain gets too bad. 5. Sleep with the head of the bed on blocks.

2. Use a stool softener or bulk laxative regularly. 3. Use correct insulin injection technique. 4. Take the pain medication before the pain gets too bad. 1. The client is being discharged. There is no need for the client to continue record- ing intake and output at home. 2. The client has undergone a radical and extensive surgery and will need nar- cotic pain medication, and a bowel regimen should be in place to prevent constipation. 3. Removal of the pancreas will create a diabetic state for the client. The client will need insulin and pancreatic enzyme replacement. 4. The client should not allow pain to reach above a "5" before taking pain medication or it will be more difficult to get the pain under control. 5. There is no reason for the client to sleep with the head of the bed elevated.

The 30-year-old female client is admitted with complaints of numbness, tingling, a crawling sensation affecting the extremities, and double vision which has occurred two (2) times in the month. Which question is most important for the nurse to ask the client? 1. "Have you experienced any difficulty with your menstrual cycle?" 2. "Have you noticed a rash across the bridge of your nose?" 3. "Do you get tired easily and sometimes have problems swallowing?" 4. "Are you taking birth control pills to prevent conception?"

3. "Do you get tired easily and sometimes have problems swallowing?" These are clinical manifestation of MS and can go undiagnosed for years be- cause of the remitting-relapsing nature of the disease. Fatigue and difficulty swallowing are other symptoms of MS.

The client presents with a complete blockage of the large intestine from a tumor. Which health-care provider's order would the nurse question? 1. Obtain consent for a colonoscopy and biopsy. 2. Start an IV of 0.9% saline at 125 mL/hr. 3. Administer 3 liters of GoLYTELY. 4. Give tap water enemas until it is clear.

3. Administer 3 liters of GoLYTELY. This client has an intestinal blockage from a solid tumor blocking the colon. Although the client needs to be cleaned out for the colonoscopy, GoLYTELY could cause severe cramping without a reasonable benefit to the client and could cause a medical emergency.

The male client diagnosed with chronic pancreatitis calls and reports to the clinic nurse he has been having a lot of "gas," along with frothy and very foul-smelling stools. Which intervention should the nurse implement? 1. Explain this is common for chronic pancreatitis. 2. Ask the client to bring in a stool specimen to the clinic. 3. Arrange an appointment with the HCP for today. 4. Discuss the need to decrease fat in the diet so this won't happen.

3. Arrange an appointment with the HCP for today. Steatorrhea (fatty, frothy, foul-smelling stool) is caused by a decrease in pan- creatic enzyme secretion and indicates impaired digestion and possibly an increase in the severity of the pancre- atitis. The client should see the HCP.

The client is diagnosed with ALS. As the disease progresses, which intervention should the nurse implement? 1. Discuss the need to be placed in a long-term care facility. 2. Explain how to care for a sigmoid colostomy. 3. Assist the client to prepare an advance directive. 4. Teach the client how to use a motorized wheelchair.

3. Assist the client to prepare an advance directive. A client with ALS usually dies within five (5) years. Therefore, the nurse should offer the client the opportunity to determine how he/she wants to die

The nurse is completing discharge teaching to the client diagnosed with acute pancreatitis. Which instruction should the nurse discuss with the client? 1. Instruct the client to decrease alcohol intake. 2. Explain the need to avoid all stress. 3. Discuss the importance of stopping smoking. 4. Teach the correct way to take pancreatic enzymes.

3. Discuss the importance of stopping smoking. 3. Smoking stimulates the pancreas to release pancreatic enzymes and should be stopped.

Which disease is the client diagnosed with GERD at greater risk for developing? 1. Hiatal hernia. 2. Gastroenteritis. 3. Esophageal cancer. 4. Gastric cancer.

3. Esophageal cancer. Barrett's esophagus results from long- term erosion of the esophagus as a result of reflux of stomach contents secondary to GERD. This is a precursor to esophageal cancer.

The nurse is caring for clients in an outpatient clinic. Which information should the nurse teach regarding the American Cancer Society's recommendations for the early detection of colon cancer? 1. Beginning at age 60, a digital rectal examination should be done yearly. 2. After reaching middle age, a yearly fecal occult blood test should be done. 3. Have a colonoscopy at age 50 and then once every five (5) to 10 years. 4. A flexible sigmoidoscopy should be done yearly after age 40.

3. Have a colonoscopy at age 50 and then once every five (5) to 10 years. The American Cancer Society recom- mends a colonoscopy at age 50 and every five (5) to 10 years thereafter, and a flexible sigmoidoscopy and a barium enema every five (5) years.

The nurse is planning a program for clients at a health fair regarding the prevention and early detection of cancer of the pancreas. Which self-care activity should the nurse discuss which is an example of a primary nursing intervention? 1. Monitor for elevated blood glucose at random intervals. 2. Inspect the skin and sclera of the eyes for a yellow tint. 3. Limit meat in the diet and eat a diet low in fat. 4. Instruct the client with hyperglycemia about insulin injections.

3. Limit meat in the diet and eat a diet low in fat. Limiting the intake of meat and fats in the diet is an example of primary inter- ventions. Risk factors for the develop- ment of cancer of the pancreas are cigarette smoking and eating a high-fat diet. By changing these behaviors, the client could possibly prevent the devel- opment of cancer of the pancreas. Other risk factors include genetic pre- disposition and exposure to industrial chemicals.

The client diagnosed with an acute exacerbation of multiple sclerosis is placed on high-dose intravenous injections of corticosteroid medication. Which nursing intervention should be implemented? 1. Discuss discontinuing the proton pump inhibitor with the HCP. 2. Hold the medication until after all cultures have been obtained. 3. Monitor the client's serum blood glucose levels frequently. 4. Provide supplemental dietary sodium with the client's meals.

3. Monitor the client's serum blood glucose levels frequently. 3. Steroids interfere with glucose metab- olism by blocking the action of insulin; therefore, the blood glucose levels should be monitored.

The nurse is discussing complications of chronic pancreatitis with a client diagnosed with the disease. Which complication should the nurse discuss with the client? 1. Diabetes insipidus. 2. Crohn's disease. 3. Narcotic addiction. 4. Peritonitis.

3. Narcotic addiction. Narcotic addiction is related to the frequent, severe pain episodes often occurring with chronic pancreatitis which require narcotics for relief.

The client admitted to rule out pancreatic islet tumors complains of feeling weak, shaky, and sweaty. Which priority intervention should be implemented by the nurse? 1. Start an IV with D5W. 2. Notify the health-care provider. 3. Perform a bedside glucose check. 4. Give the client some orange juice.

3. Perform a bedside glucose check. These are symptoms of an insulin reac- tion (hypoglycemia). A bedside glucose check should be done. Pancreatic islet tumors can produce hyperinsulinemia or hypoglycemia.

Which task is most appropriate for the nurse to delegate to the unlicensed assistive personnel (UAP)? 1. Draw the serum liver function test. 2. Evaluate the client's intake and output. 3. Perform the bedside glucometer check. 4. Help the ward clerk transcribe orders.

3. Perform the bedside glucometer check. The UAP can perform a bedside glucometer check, but the nurse must evaluate the result and determine any action needed.

The client is admitted to the medical department with a diagnosis of R/O acute pancreatitis. Which laboratory values should the nurse monitor to confirm this diagnosis? 1. Creatinine and BUN. 2. Troponin and CK-MB. 3. Serum amylase and lipase. 4. Serum bilirubin and calcium.

3. Serum amylase and lipase. Serum amylase increases within two (2) to 12 hours of the onset of acute pan- creatitis to two (2) to three (3) times normal and returns to normal in three (3) to four (4) days; lipase elevates and remains elevated for seven (7) to 14 days.

The client diagnosed with acute pancreatitis is in pain. Which position should the nurse assist the client to assume to help decrease the pain? 1. Recommend lying in the prone position with legs extended. 2. Maintain a tripod position over the bedside table. 3. Side-lying with knees flexed 4. Encourage a supine position with a pillow under the knees.

3. Side-lying with knees flexed 3. This fetal position decreases pain caused by the stretching of the peritoneum as a result of edema.

The home health nurse is caring for the client newly diagnosed with multiple sclerosis. Which client issue is of most importance? 1. The client refuses to have a gastrostomy feeding. 2. The client wants to discuss if she should tell her fiancé. 3. The client tells the nurse life is not worth living anymore. 4. The client needs the flu and pneumonia vaccines.

3. The client tells the nurse life is not worth living anymore. 3. A potential suicide statement is priority for the nurse when caring for the client with MS.

Which statement by the client diagnosed with hepatitis warrants immediate intervention by the clinic nurse? 1. "I will not drink any type of beer or mixed drink." 2. "I will get adequate rest so I don't get exhausted." 3. "I had a big hearty breakfast this morning." 4. "I took some cough syrup for this nasty head cold."

4. "I took some cough syrup for this nasty head cold." 4. The client needs to understand some types of cough syrup have alcohol and all alcohol must be avoided to prevent further injury to the liver; therefore, this statement requires intervention.

The 45-year-old client is diagnosed with primary progressive multiple sclerosis and the nurse writes the nursing diagnosis "anticipatory grieving related to progressive loss." Which intervention should be implemented? 1. Consult the physical therapist for assistive devices for mobility. 2. Determine if the client has a legal power of attorney. 3. Ask if the client would like to talk to the hospital chaplain. 4. Discuss the client's wishes regarding end-of-life care.

4. Discuss the client's wishes regarding end-of-life care. The client should make personal choices about end-of-life issues while it is possi- ble to do so. This client is progressing toward immobility and all the complica- tions related to it.

The nurse is caring for a client diagnosed with GERD. Which nursing interventions should be implemented? 1. Place the client prone in bed and administer nonsteroidal anti-inflammatory medications. 2. Have the client remain upright at all times and walk for 30 minutes three (3) times a week. 3. Instruct the client to maintain a right lateral side-lying position and take antacids before meals. 4. Elevate the head of the bed 30 degrees and discuss lifestyle modifications with the client.

4. Elevate the head of the bed 30 degrees and discuss lifestyle modifications with the client. The head of the bed should be elevated to allow gravity to help in preventing reflux. Lifestyle modifications of losing weight, making dietary modifications, attempting smoking cessation, discon- tinuing the use of alcohol, and not stooping or bending at the waist all help to decrease reflux.

The client who has had an abdominal perineal resection is being discharged. Which discharge information should the nurse teach? 1. The stoma should be a white, blue, or purple color. 2. Limit ambulation to prevent the pouch from coming off. 3. Take pain medication when the pain level is at an "8." 4. Empty the pouch when it is one-third to one-half full.

4. Empty the pouch when it is one-third to one-half full. The pouch should be emptied when it is one-third to one-half full to prevent the contents from becoming too heavy for the seal to hold and to prevent leakage from occurring.

The nurse caring for a client diagnosed with cancer of the pancreas writes the nursing diagnosis of "risk for altered skin integrity related to pruritus." Which intervention should the nurse implement? 1. Assess tissue turgor. 2. Apply antifungal creams. 3. Monitor bony prominences for breakdown. 4. Have the client keep the fingernails short.

4. Have the client keep the fingernails short. Keeping the fingernails short will reduce the chance of breaks in the skin from scratching.

The client is diagnosed with acute pancreatitis. Which health-care provider's admitting order should the nurse question? 1. Bedrest with bathroom privileges. 2. Initiate IV therapy of D5W at 125 mL/hr. 3. Weigh client daily. 4. Low-fat, low-carbohydrate diet.

4. Low-fat, low-carbohydrate diet. The client will be NPO, which will decrease stimulation of the pancreatic enzymes, resulting in decreased autodigestion of the pancreas, therefore decreasing pain.

The nurse caring for a client diagnosed with cancer of the pancreas writes the problem of "altered nutrition: less than body requirements." Which collaborative intervention should the nurse include in the plan of care? 1. Continuous feedings via PEG tube. 2. Have the family bring in foods from home. 3. Assess for food preferences. 4. Refer to the dietitian.

4. Refer to the dietitian. A collaborative intervention is to refer to the nutrition expert, the dietitian.

The client diagnosed with ALS is prescribed an antiglutamate, riluzole (Rilutek). Which instruction should the nurse discuss with the client? 1. Take the medication with food. 2. Do not eat green, leafy vegetables. 3. Use SPF 30 when going out in the sun. 4. Report any febrile illness.

4. Report any febrile illness The medication can cause blood dyscrasias. Therefore, the client is monitored for liver function, blood count, blood chemistries, and alkaline phosphatase. The client should report any febrile illness. This is the first medication developed to treat ALS.

The school nurse is discussing methods to prevent an outbreak of hepatitis A with a group of high school teachers. Which action is the most important to teach the high school teachers? 1. Do not allow students to eat or drink after each other. 2. Drink bottled water as much as possible. 3. Encourage protected sexual activity. 4. Sing the happy birthday song while washing hands.

4. Sing the happy birthday song while washing hands. Hepatitis A is transmitted via the fecal-oral route. Good hand washing helps to prevent its spread. Singing the happy birthday song takes approximately 30 seconds, which is how long an indi- vidual should wash his or her hands.

The nurse and a licensed practical nurse (LPN) are caring for a group of clients. Which nursing task should not be assigned to the LPN? 1. Administer a skeletal muscle relaxant to a client diagnosed with low back pain. 2. Discuss bowel regimen medications with the HCP for the client on strict bedrest. 3. Draw morning blood work on the client diagnosed with bacterial meningitis. 4. Teach self-catheterization to the client diagnosed with multiple sclerosis.

4. Teach self-catheterization to the client diagnosed with multiple sclerosis. The nurse should not assign assessing, teaching, or evaluation to the LPN. Evaluating the client's ability to per- form self-catheterization should not be assigned to the LPN.

The nurse writes a psychosocial problem of "risk for altered sexual functioning related to new colostomy." Which intervention should the nurse implement? 1. Tell the client there should be no intimacy for at least three (3) months. 2. Ensure the client and significant other are able to change the ostomy pouch. 3. Demonstrate with charts possible sexual positions for the client to assume. 4. Teach the client to protect the pouch from becoming dislodged during sex.

4. Teach the client to protect the pouch from becoming dislodged during sex. A pouch that becomes dislodged during the sexual act would cause embarrass- ment for the client, whose body image has already been dealt a blow.

The nurse is assessing a 48-year-old client diagnosed with multiple sclerosis. Which clinical manifestation warrants immediate intervention? 1. The client has scanning speech and diplopia. 2. The client has dysarthria and scotomas. 3. The client has muscle weakness and spasticity. 4. The client has a congested cough and dysphagia.

4. The client has a congested cough and dysphagia. Dysphagia is a common problem of clients diagnosed with multiple sclerosis, and this places the client at risk for aspi- ration pneumonia. Some clients diag- nosed with multiple sclerosis eventually become immobile and are at risk for pneumonia.

The client is being evaluated to rule out ALS. Which signs/symptoms would the nurse note to confirm the diagnosis? 1. Muscle atrophy and flaccidity. 2. Fatigue and malnutrition. 3. Slurred speech and dysphagia. 4. Weakness and paralysis.

4. Weakness and paralysis. ALS results from the degeneration and demyelination of motor neurons in the spinal cord, which results in paralysis and weakness of the muscles.

An adult patient has been admitted to the medical unit for the treatment of acute pancreatitis. What nursing action should be included in this patients plan of care? A) Measure the patients abdominal girth daily. B) Limit the use of opioid analgesics. C) Monitor the patient for signs of dysphagia. D) Encourage activity as tolerated.

A) Measure the patients abdominal girth daily.

The nurse is caring for a patient with a large venous leg ulcer. What intervention should the nurse implement to promote healing and prevent infection? A) Provide a high-calorie, high-protein diet. B) Apply a clean occlusive dressing once daily and whenever soiled. C) Irrigate the wound with hydrogen peroxide once daily. D) Apply an antibiotic ointment on the surrounding skin with each dressing change.

A) Provide a high-calorie, high-protein diet.

You are developing a care plan for a patient with Cushing syndrome. What nursing diagnosis would have the highest priority in this care plan? A) Risk for injury related to weakness B) Ineffective breathing pattern related to muscle weakness C) Risk for loneliness related to disturbed body image D) Autonomic dysreflexia related to neurologic changes

A) Risk for injury related to weakness

The nurse is caring for a 77-year-old woman with MS. She states that she is very concerned about the progress of her disease and what the future holds. The nurse should know that elderly patients with MS are known to be particularly concerned about what variables? Select all that apply. A) Possible nursing home placement B) Pain associated with physical therapy C) Increasing disability D) Becoming a burden on the family E) Loss of appetite

A, C, D

A patients physician has ordered a liver panel in response to the patients development of jaundice. When reviewing the results of this laboratory testing, the nurse should expect to review what blood tests? Select all that apply. A) Alanine aminotransferase (ALT) B) C-reactive protein (CRP) C) Gamma-glutamyl transferase (GGT) D) Aspartate aminotransferase (AST) E) B-type natriuretic peptide (BNP)

A) Alanine aminotransferase (ALT) C) Gamma-glutamyl transferase (GGT) D) Aspartate aminotransferase (AST)

A nurse is caring for a patient with hepatic encephalopathy. While making the initial shift assessment, the nurse notes that the patient has a flapping tremor of the hands. The nurse should document the presence of what sign of liver disease? A) Asterixis B) Constructional apraxia C) Fetor hepaticus D) Palmar erythema

A) Asterixis

A nurse is creating an education plan for a patient with venous insufficiency. What measure should the nurse include in the plan? A) Avoiding tight-fitting socks. B) Limit activity whenever possible. C) Sleep with legs in a dependent position. D) Avoid the use of pressure stockings.

A) Avoiding tight-fitting socks.

A patient with a liver mass is undergoing a percutaneous liver biopsy. What action should the nurse perform when assisting with this procedure? A) Position the patient on the right side with a pillow under the costal margin after the procedure. B) Administer 1 unit of albumin 90 minutes before the procedure as ordered. C) Administer at least 1 unit of packed red blood cells as ordered the day before the scheduled procedure. D) Confirm that the patients electrolyte levels have been assessed prior to the procedure.

A) Position the patient on the right side with a pillow under the costal margin after the procedure.

The clinic nurse is caring for a 57-year-old client who reports experiencing leg pain whenever she walks several blocks. The patient has type 1 diabetes and has smoked a pack of cigarettes every day for the past 40 years. The physician diagnoses intermittent claudication. The nurse should provide what instruction about long-term care to the client? A) Be sure to practice meticulous foot care. B) Consider cutting down on your smoking. C) Reduce your activity level to accommodate your limitations. D) Try to make sure you eat enough protein.

A) Be sure to practice meticulous foot care. Intermittent claudication and other chronic peripheral vascular diseases reduce oxygenation to the feet, making them susceptible to injury and poor healing; therefore, meticulous foot care is essential.

A patient with Cushing syndrome as a result of a pituitary tumor has been admitted for a transsphenoidal hypophysectomy. What would be most important for the nurse to monitor before, during, and after surgery? A) Blood glucose B) Assessment of urine for blood C) Weight D) Oral temperature

A) Blood glucose

When assessing venous disease in a patients lower extremities, the nurse knows that what test will most likely be ordered? A) Duplex ultrasonography B) Echocardiography C) Positron emission tomography (PET) D) Radiography

A) Duplex ultrasonography

The nurse is caring for a patient who is hospitalized with an exacerbation of MS. To ensure the patients safety, what nursing action should be performed? A) Ensure that suction apparatus is set up at the bedside. B) Pad the patients bed rails. C) Maintain bed rest whenever possible. D) Provide several small meals each day.

A) Ensure that suction apparatus is set up at the bedside.

A patient with advanced venous insufficiency is confined following orthopedic surgery. How can the nurse best prevent skin breakdown in the patients lower extremities? A) Ensure that the patients heels are protected and supported. B) Closely monitor the patients serum albumin and prealbumin levels. C) Perform gentle massage of the patients lower legs, as tolerated. D) Perform passive range-of-motion exercises once per shift.

A) Ensure that the patients heels are protected and supported.

Following an addisonian crisis, a patients adrenal function has been gradually regained. The nurse should ensure that the patient knows about the need for supplementary glucocorticoid therapy in which of the following circumstances? A) Episodes of high psychosocial stress B) Periods of dehydration C) Episodes of physical exertion D) Administration of a vaccine

A) Episodes of high psychosocial stress

A patient with MS has been admitted to the hospital following an acute exacerbation. When planning the patients care, the nurse addresses the need to enhance the patients bladder control. What aspect of nursing care is most likely to meet this goal? A) Establish a timed voiding schedule. B) Avoid foods that change the pH of urine. C) Perform intermittent catheterization q6h. D) Administer anticholinergic drugs as ordered.

A) Establish a timed voiding schedule.

The nurse providing care for a patient with Cushing syndrome has identified the nursing diagnosis of risk for injury related to weakness. How should the nurse best reduce this risk? A) Establish falls prevention measures. B) Encourage bed rest whenever possible. C) Encourage the use of assistive devices. D) Provide constant supervision.

A) Establish falls prevention measures.

During a health education session, a participant has asked about the hepatitis E virus. What prevention measure should the nurse recommend for preventing infection with this virus? A) Following proper hand-washing techniques B) Avoiding chemicals that are toxic to the liver C) Wearing a condom during sexual contact D) Limiting alcohol intake

A) Following proper hand-washing techniques

A triage nurse in the emergency department is assessing a patient who presented with complaints of general malaise. Assessment reveals the presence of jaundice and increased abdominal girth. What assessment question best addresses the possible etiology of this patients presentation? A) How many alcoholic drinks do you typically consume in a week? B) To the best of your knowledge, are your immunizations up to date? C) Have you ever worked in an occupation where you might have been exposed to toxins? D) Has anyone in your family ever experienced symptoms similar to yours?

A) How many alcoholic drinks do you typically consume in a week?

A patients assessment and diagnostic testing are suggestive of acute pancreatitis. When the nurse is performing the health interview, what assessment questions address likely etiologic factors? Select all that apply. A) How many alcoholic drinks do you typically consume in a week? B) Have you ever been tested for diabetes? C) Have you ever been diagnosed with gallstones? D) Would you say that you eat a particularly high-fat diet? E) Does anyone in your family have cystic fibrosis?

A) How many alcoholic drinks do you typically consume in a week? C) Have you ever been diagnosed with gallstones?

A nurse educator is teaching a group of recent nursing graduates about their occupational risks for contracting hepatitis B. What preventative measures should the educator promote? Select all that apply. A) Immunization B) Use of standard precautions C) Consumption of a vitamin-rich diet D) Annual vitamin K injections E) Annual vitamin B12 injections

A) Immunization B) Use of standard precautions

A patient presents at the walk-in clinic complaining of diarrhea and vomiting. The patient has a documented history of adrenal insufficiency. Considering the patients history and current symptoms, the nurse should anticipate that the patient will be instructed to do which of the following? A) Increase his intake of sodium until the GI symptoms improve. B) Increase his intake of potassium until the GI symptoms improve. C) Increase his intake of glucose until the GI symptoms improve. D) Increase his intake of calcium until the GI symptoms improve.

A) Increase his intake of sodium until the GI symptoms improve. The patient will need to supplement dietary intake with added salt during episodes of GI losses of fluid through vomiting and diarrhea to prevent the onset of addisonian crisis.

The nurse is working with a patient who is newly diagnosed with MS. What basic information should the nurse provide to the patient? A) MS is a progressive demyelinating disease of the nervous system. B) MS usually occurs more frequently in men. C) MS typically has an acute onset. D) MS is sometimes caused by a bacterial infection.

A) MS is a progressive demyelinating disease of the nervous system.

A 37-year-old male patient presents at the emergency department (ED) complaining of nausea and vomiting and severe abdominal pain. The patients abdomen is rigid, and there is bruising to the patients flank. The patients wife states that he was on a drinking binge for the past 2 days. The ED nurse should assist in assessing the patient for what health problem? A) Severe pancreatitis with possible peritonitis B) Acute cholecystitis C) Chronic pancreatitis D) Acute appendicitis with possible perforation

A) Severe pancreatitis with possible peritonitis

A patient with gastroesophageal reflux disease (GERD) has a diagnosis of Barretts esophagus with minor cell changes. Which of the following principles should be integrated into the patients subsequent care? A) The patient will require an upper endoscopy every 6 months to detect malignant changes. B) Liver enzymes must be checked regularly, as H2 receptor antagonists may cause hepatic damage. C) Small amounts of blood are likely to be present in the stools and are not cause for concern. D) Antacids may be discontinued when symptoms of heartburn subside.

A) The patient will require an upper endoscopy every 6 months to detect malignant changes.

A patient with suspected adrenal insufficiency has been ordered an adrenocorticotropic hormone (ACTH) stimulation test. Administration of ACTH caused a marked increase in cortisol levels. How should the nurse interpret this finding? A) The patients pituitary function is compromised. B) The patients adrenal insufficiency is not treatable. C) The patient has insufficient hypothalamic function. D) The patient would benefit from surgery.

A) The patients pituitary function is compromised.

A patient is undergoing testing for suspected adrenocortical insufficiency. The care team should ensure that the patient has been assessed for the most common cause of adrenocortical insufficiency. What is the most common cause of this health problem? A) Therapeutic use of corticosteroids B) Pheochromocytoma C) Inadequate secretion of ACTH D) Adrenal tumor

A) Therapeutic use of corticosteroids

A patient has developed hepatic encephalopathy secondary to cirrhosis and is receiving care on the medical unit. The patients current medication regimen includes lactulose (Cephulac) four times daily. What desired outcome should the nurse relate to this pharmacologic intervention? A) Two to 3 soft bowel movements daily B) Significant increase in appetite and food intake C) Absence of nausea and vomiting D) Absence of blood or mucus in stool

A) Two to 3 soft bowel movements daily

A nurse cares for a client who presents with tachycardia and prostration related to biliary colic. Which actions should the nurse take? (Select all that apply.) a. Contact the provider immediately. b. Lower the head of the bed. c. Decrease intravenous fluids. d. Ask the client to bear down. e. Administer prescribed opioids.

A, B Clients who are experiencing biliary colic may present with tachycardia, pallor, diaphoresis, prostration, or other signs of shock.

A client has been discharged to an inpatient rehabilitation center after an esophagogastrectomy. What menu selections by the client at the rehabilitation center indicate a good understanding of dietary instructions? (Select all that apply.) a. Boost supplement b. Greek yogurt c. Scrambled eggs d. Whole milk shake e. Whole wheat toast

A, B, C, D

The nurse is aware that which factors are related to the development of gastroesophageal reflux disease (GERD)? (Select all that apply.) a. Delayed gastric emptying b. Eating large meals c. Hiatal hernia d. Obesity e. Viral infections

A, B, C, D

An adult patient has sought care for the treatment of headaches that have become increasingly severe and frequent over the past several months. Which of the following questions addresses potential etiological factors? Select all that apply? A) Are you exposed to any toxins or chemicals at work? B) How would you describe your ability to cope with stress? C) What medications are you currently taking? D) When was the last time you were hospitalized? E) Does anyone else in your family struggle with headaches?

A, B, C, E

A nurse assesses a client who is experiencing a cluster headache. Which clinical manifestations should the nurse expect to find? (Select all that apply.) a. Ipsilateral tearing of the eye b. Miosis c. Abrupt loss of consciousness d. Neck and shoulder tenderness e. Nasal congestion f. Exophthalmos

A, B, E

A nurse assesses a male client who has symptoms of cirrhosis. Which questions should the nurse ask to identify potential factors contributing to this laboratory result? (Select all that apply.) a. How frequently do you drink alcohol? b. Have you ever had sex with a man? c. Do you have a family history of cancer? d. Have you ever worked as a plumber? e. Were you previously incarcerated?

A, B, E

A nurse is preparing to administer pantoprazole (Protonix) intravenously. What actions by the nurse are most appropriate? (Select all that apply.) a. Administer the drug through a separate IV line. b. Infuse pantoprazole using an IV pump. c. Keep the drug in its original brown bag. d. Take vital signs frequently during infusion. e. Use an in-line IV filter when infusing.

A, B, E

A nurse delegates hygiene care for a client who has advanced cirrhosis to an unlicensed nursing personnel (UAP). Which statements should the nurse include when delegating this task to the UAP? (Select all that apply.) a. Apply lotion to the clients dry skin areas. b. Use a basin with warm water to bathe the client. c. For the clients oral care, use a soft toothbrush. d. Provide clippers so the client can trim the fingernails. e. Bathe with antibacterial and water-based soaps.

A, C, D

A nurse plans care for a client who has hepatopulmonary syndrome. Which interventions should the nurse include in this clients plan of care? (Select all that apply.) a. Oxygen therapy b. Prone position c. Feet elevated on pillows d. Daily weights e. Physical therapy

A, C, D

A patient with Cushing syndrome has been hospitalized after a fall. The dietician consulted works with the patient to improve the patients nutritional intake. What foods should a patient with Cushing syndrome eat to optimize health? Select all that apply. A) Foods high in vitamin D B) Foods high in calories C) Foods high in protein D) Foods high in calcium E) Foods high in sodium

A, C, D

A nurse is teaching clients with gastroesophageal reflux disease (GERD) about foods to avoid. Which foods should the nurse include in the teaching? (Select all that apply.) a. Chocolate b. Decaffeinated coffee c. Citrus fruits d. Peppermint e. Tomato sauce

A, C, D, E

An infection control nurse develops a plan to decrease the number of health care professionals who contract viral hepatitis at work. Which ideas should the nurse include in this plan? (Select all that apply.) a. Policies related to consistent use of Standard Precautions b. Hepatitis vaccination mandate for workers in high-risk areas c. Implementation of a needleless system for intravenous therapy d. Number of sharps used in client care reduced where possible e. Postexposure prophylaxis provided in a timely manner

A, C, D, E

The nurse has performed a thorough nursing assessment of the care of a patient with chronic leg ulcers. The nurses assessment should include which of the following components? Select all that apply. A) Location and type of pain B) Apical heart rate C) Bilateral comparison of peripheral pulses D) Comparison of temperature in the patients legs E) Identification of mobility limitations

A, C, D, E

A nurse is caring for a patient in the late stages of esophageal cancer. The nurse should plan to prevent or address what characteristics of this stage of the disease? Select all that apply. A) Perforation into the mediastinum B) Development of an esophageal lesion C) Erosion into the great vessels D) Painful swallowing E) Obstruction of the esophagus

A, C, E

A nurse plans care for a client who has acute pancreatitis and is prescribed nothing by mouth (NPO). With which health care team members should the nurse collaborate to provide appropriate nutrition to this client? (Select all that apply.) a. Registered dietitian b. Nursing assistant c. Clinical pharmacist d. Certified herbalist e. Health care provider

A, C, E

A patient with end-stage liver disease has developed hypervolemia. What nursing interventions would be most appropriate when addressing the patients fluid volume excess? Select all that apply. A) Administering diuretics B) Administering calcium channel blockers C) Implementing fluid restrictions D) Implementing a 1500 kcal/day restriction E) Enhancing patient positioning

A, C, E

The nurse is caring for a patient with a diagnosis of Addisons disease. What sign or symptom is most closely associated with this health problem? A) Truncal obesity B) Hypertension C) Muscle weakness D) Moon face

C) Muscle weakness Patients with Addisons disease demonstrate muscular weakness, anorexia, gastrointestinal symptoms, fatigue, emaciation, dark pigmentation of the skin, and hypotension. Patients with Cushing syndrome demonstrate truncal obesity, moon face, acne, abdominal striae, and hypertension.

The nurse is caring for a patient with multiple sclerosis (MS). The patient tells the nurse the hardest thing to deal with is the fatigue. When teaching the patient how to reduce fatigue, what action should the nurse suggest? A) Taking a hot bath at least once daily B) Resting in an air-conditioned room whenever possible C) Increasing the dose of muscle relaxants D) Avoiding naps during the day

B) Resting in an air-conditioned room whenever possible

The nurse is analyzing a rhythm strip. What component of the ECG corresponds to the resting state of the patients heart? A) P wave B) T wave C) U wave D) QRS complex

B) T wave The T wave specifically represents ventricular repolarization

A patient with a history of injection drug use has been diagnosed with hepatitis C. When collaborating with the care team to plan this patients treatment, the nurse should anticipate what intervention? A) Administration of immune globulins B) A regimen of antiviral medications C) Rest and watchful waiting Administration of fresh-frozen plasma (FFP)

B) A regimen of antiviral medications

The nurse is caring for a 72-year-old patient who is in cardiac rehabilitation following heart surgery. The patient has been walking on a regular basis for about a week and walks for 15 minutes 3 times a day. The patient states that he is having a cramp-like pain in the legs every time he walks and that the pain gets better when I rest. The patients care plan should address what problem? A) Decreased mobility related to VTE B) Acute pain related to intermittent claudication C) Decreased mobility related to venous insufficiency D) Acute pain related to vasculitis

B) Acute pain related to intermittent claudication

A patient with pericarditis has just been admitted to the CCU. The nurse planning the patients care should prioritize what nursing diagnosis? A) Anxiety related to pericarditis B) Acute pain related to pericarditis C) Ineffective tissue perfusion related to pericarditis D) Ineffective breathing pattern related to pericarditis

B) Acute pain related to pericarditis The most characteristic symptom of pericarditis is chest pain, although pain also may be located beneath the clavicle, in the neck, or in the left trapezius (scapula) region. The pain or discomfort usually remains fairly constant, but it may worsen with deep inspiration and when lying down or turning.

A clinic nurse is caring for a patient diagnosed with migraine headaches. During the patient teaching session, the patient questions the nurse regarding alcohol consumption. What would the nurse be correct in telling the patient about the effects of alcohol? A) Alcohol causes hormone fluctuations. B) Alcohol causes vasodilation of the blood vessels. C) Alcohol has an excitatory effect on the CNS. D) Alcohol diminishes endorphins in the brain.

B) Alcohol causes vasodilation of the blood vessels.

A nurse is caring for a patient who experiences debilitating cluster headaches. The patient should be taught to take appropriate medications at what point in the course of the onset of a new headache? A) As soon as the patients pain becomes unbearable B) As soon as the patient senses the onset of symptoms C) Twenty to 30 minutes after the onset of symptoms D) When the patient senses his or her symptoms peaking

B) As soon as the patient senses the onset of symptoms A migraine or a cluster headache in the early phase requires abortive medication therapy instituted as soon as possible.

A nurse is caring for a patient who has been admitted for the treatment of advanced cirrhosis. What assessment should the nurse prioritize in this patients plan of care? A) Measurement of abdominal girth and body weight B) Assessment for variceal bleeding C) Assessment for signs and symptoms of jaundice D) Monitoring of results of liver function testing

B) Assessment for variceal bleeding

A patient converts from normal sinus rhythm at 80 bpm to atrial fibrillation with a ventricular response at 166 bpm. Blood pressure is 162/74 mm Hg. Respiratory rate is 20 breaths per minute with normal chest expansion and clear lungs bilaterally. IV heparin and Cardizem are given. The nurse caring for the patient understands that the main goal of treatment is what? A) Decrease SA node conduction B) Control ventricular heart rate C) Improve oxygenation D) Maintain anticoagulation

B) Control ventricular heart rate This is a priority because it directly affects cardiac output.

New nurses on the telemetry unit have been paired with preceptors. One new nurse asks her preceptor to explain depolarization. What would be the best answer by the preceptor? A) Depolarization is the mechanical contraction of the heart muscles. B) Depolarization is the electrical stimulation of the heart muscles. C) Depolarization is the electrical relaxation of the heart muscles. D) Depolarization is the mechanical relaxation of the heart muscles.

B) Depolarization is the electrical stimulation of the heart muscles. The electrical stimulation of the heart is called depolarization, and the mechanical contraction is called systole.

A nurse has written a plan of care for a man diagnosed with peripheral arterial insufficiency. One of the nursing diagnoses in the care plan is altered peripheral tissue perfusion related to compromised circulation. What is the most appropriate intervention for this diagnosis? A) Elevate his legs and arms above his heart when resting. B) Encourage the patient to engage in a moderate amount of exercise. C) Encourage extended periods of sitting or standing. D) Discourage walking in order to limit pain.

B) Encourage the patient to engage in a moderate amount of exercise.

The nurse is auscultating the breath sounds of a patient with pericarditis. What finding is most consistent with this diagnosis? A) Wheezes B) Friction rub C) Fine crackles D) Coarse crackles

B) Friction rub

A patient is brought to the emergency department by the paramedics. The patient is a type 2 diabetic and is experiencing HHS. The nurse should identify what components of HHS? Select all that apply. A) Leukocytosis B) Glycosuria C) Dehydration D) Hypernatremia E) Hyperglycemia

B) Glycosuria C) Dehydration D) Hypernatremia E) Hyperglycemia

The nurse is caring for a patient who is admitted to the medical unit for the treatment of a venous ulcer in the area of her lateral malleolus that has been unresponsive to treatment. What is the nurse most likely to find during an assessment of this patients wound? A) Hemorrhage B) Heavy exudate C) Deep wound bed D) Pale-colored wound bed

B) Heavy exudate Venous ulcerations in the area of the medial or lateral malleolus (gaiter area) are typically large, superficial, and highly exudative.

While assisting with the surgical removal of an adrenal tumor, the OR nurse is aware that the patients vital signs may change upon manipulation of the tumor. What vital sign changes would the nurse expect to see? A) Hyperthermia and tachypnea B) Hypertension and heart rate changes C) Hypotension and hypothermia D) Hyperthermia and bradycardia

B) Hypertension and heart rate changes

While assessing a patient the nurse notes that the patients ankle-brachial index (ABI) of the right leg is 0.40. How should the nurse best respond to this assessment finding? A) Assess the patients use of over-the-counter dietary supplements. B) Implement interventions relevant to arterial narrowing. C) Encourage the patient to increase intake of foods high in vitamin K. D) Adjust the patients activity level to accommodate decreased coronary output.

B) Implement interventions relevant to arterial narrowing. ABI is used to assess the degree of stenosis of peripheral arteries. An ABI of less than 1.0 indicates possible claudication of the peripheral arteries.

The nurse is taking a health history of a new patient. The patient reports experiencing pain in his left lower leg and foot when walking. This pain is relieved with rest. The nurse notes that the left lower leg is slightly edematous and is hairless. When planning this patients subsequent care, the nurse should most likely address what health problem? A) Coronary artery disease (CAD) B) Intermittent claudication C) Arterial embolus D) Raynauds disease

B) Intermittent claudication

A patient with a diagnosis of esophageal varices has undergone endoscopy to gauge the progression of this complication of liver disease. Following the completion of this diagnostic test, what nursing intervention should the nurse perform? A) Keep patient NPO until the results of test are known. B) Keep patient NPO until the patients gag reflex returns. C) Administer analgesia until post-procedure tenderness is relieved. D) Give the patient a cold beverage to promote swallowing ability.

B) Keep patient NPO until the patients gag reflex returns.

A patient with a history of type 1 diabetes has just been admitted to the critical care unit (CCU) for diabetic ketoacidosis. The CCU nurse should prioritize what assessment during the patients initial phase of treatment? A) Monitoring the patient for dysrhythmias B) Maintaining and monitoring the patients fluid balance C) Assessing the patients level of consciousness D) Assessing the patient for signs and symptoms of venous thromboembolism

B) Maintaining and monitoring the patients fluid balance In addition to treating hyperglycemia, management of DKA is aimed at correcting dehydration, electrolyte loss, and acidosis before correcting the hyperglycemia with insulin.

The nurse is caring for a patient at risk for an addisonian crisis. For what associated signs and symptoms should the nurse monitor the patient? Select all that apply. A) Epistaxis B) Pallor C) Rapid respiratory rate D) Bounding pulse E) Hypotension

B) Pallor C) Rapid respiratory rate E) Hypotension

The nurse is caring for a patient with refractory atrial fibrillation who underwent the maze procedure several months ago. The nurse reviews the result of the patients most recent cardiac imaging, which notes the presence of scarring on the atria. How should the nurse best respond to this finding? A) Recognize that the procedure was unsuccessful. B) Recognize this as a therapeutic goal of the procedure. C) Liaise with the care team in preparation for repeating the maze procedure. D) Prepare the patient for pacemaker implantation.

B) Recognize this as a therapeutic goal of the procedure.

A patient has been diagnosed with an esophageal diverticulum after undergoing diagnostic imaging. When taking the health history, the nurse should expect the patient to describe what sign or symptom? A) Burning pain on swallowing B) Regurgitation of undigested food C) Symptoms mimicking a heart attack D) Chronic parotid abscesses

B) Regurgitation of undigested food

A nurse is caring for a patient with cirrhosis secondary to heavy alcohol use. The nurses most recent assessment reveals subtle changes in the patients cognition and behavior. What is the nurses most appropriate response? A) Ensure that the patients sodium intake does not exceed recommended levels. B) Report this finding to the primary care provider due to the possibility of hepatic encephalopathy. C) Inform the primary care provider that the patient should be assessed for alcoholic hepatitis. D) Implement interventions aimed at ensuring a calm and therapeutic care environment.

B) Report this finding to the primary care provider due to the possibility of hepatic encephalopathy.

A diabetic patient calls the clinic complaining of having a flu bug. The nurse tells him to take his regular dose of insulin. What else should the nurse tell the patient? A) Make sure to stick to your normal diet. B) Try to eat small amounts of carbs, if possible. C) Ensure that you check your blood glucose every hour. D) For now, check your urine for ketones every 8 hours.

B) Try to eat small amounts of carbs, if possible. For prevention of DKA related to illness, the patient should attempt to consume frequent small portions of carbohydrates (including foods usually avoided, such as juices, regular sodas, and gelatin).

The nurse is providing care for a patient who has just been diagnosed with peripheral arterial occlusive disease (PAD). What assessment finding is most consistent with this diagnosis? A) Numbness and tingling in the distal extremities B) Unequal peripheral pulses between extremities C) Visible clubbing of the fingers and toes D) Reddened extremities with muscle atrophy

B) Unequal peripheral pulses between extremities PAD assessment may manifest as unequal pulses between extremities, with the affected leg cooler and paler than the unaffected leg.

A nurse cares for a client with pancreatic cancer who is prescribed implanted radioactive iodine seeds. Which actions should the nurse take when caring for this client? (Select all that apply.) a. Dispose of dirty linen in a red biohazard bag. b. Place the client in a private room. c. Wear a lead apron when providing client care. d. Bundle care to minimize exposure to the client. e. Initiate Transmission-Based Precautions.

B, C, D

A nurse collaborates with an unlicensed assistive personnel (UAP) to provide care for a client who is in the healing phase of acute pancreatitis. Which statements focused on nutritional requirements should the nurse include when delegating care for this client? (Select all that apply.) a. Do not allow the client to eat between meals. b. Make sure the client receives a protein shake. c. Do not allow caffeine-containing beverages. d. Make sure the foods are bland with little spice. e. Do not allow high-carbohydrate food items.

B, C, D During the healing phase of pancreatitis, the client should be provided small, frequent, moderate- to high- carbohydrate, high-protein, low-fat meals.

A nurse assesses a client who is recovering from a Whipple procedure. Which clinical manifestations alert the nurse to a complication from this procedure? (Select all that apply.) a. Clay-colored stools b. Substernal chest pain c. Shortness of breath d. Lack of bowel sounds or flatus e. Urine output of 20 mL/6 hr

B, C, D, E

An emergency room nurse assesses a client with potential liver trauma. Which clinical manifestations should alert the nurse to internal bleeding and hypovolemic shock? (Select all that apply.) a. Hypertension b. Tachycardia c. Flushed skin d. Confusion e. Shallow respirations

B, D

A nurse is teaching a client who has chronic headaches. Which statements about headache triggers should the nurse include in this clients plan of care? (Select all that apply.) a. Increase your intake of caffeinated beverages. b. Incorporate physical exercise into your daily routine. c. Avoid all alcoholic beverages. d. Participate in a smoking cessation program. e. Increase your intake of fruits and vegetables.

B, D, E

A nurse teaches a client who is recovering from acute pancreatitis. Which statements should the nurse include in this clients teaching? (Select all that apply.) a. Take a 20-minute walk at least 5 days each week. b. Attend local Alcoholics Anonymous (AA) meetings weekly. c. Choose whole grains rather than foods with simple sugars. d. Use cooking spray when you cook rather than margarine or butter. e. Stay away from milk and dairy products that contain lactose. f. We can talk to your doctor about a prescription for nicotine patches.

B, D, F

A nurse assesses a client who has liver disease. Which laboratory findings should the nurse recognize as potentially causing complications of this disorder? (Select all that apply.) a. Elevated aspartate transaminase b. Elevated international normalized ratio (INR) c. Decreased serum globulin levels d. Decreased serum alkaline phosphatase e. Elevated serum ammonia f. Elevated prothrombin time (PT)

B, E, F

A 79-year-old man is admitted to the medical unit with digital gangrene. The man states that his problems first began when he stubbed his toe going to the bathroom in the dark. In addition to this trauma, the nurse should suspect that the patient has a history of what health problem? A) Raynauds phenomenon B) CAD C) Arterial insufficiency D) Varicose veins

C) Arterial insufficiency

A patient with mitral stenosis exhibits new symptoms of a dysrhythmia. Based on the pathophysiology of this disease process, the nurse would expect the patient to exhibit what heart rhythm? A) Ventricular fibrillation (VF) B) Ventricular tachycardia (VT) C) Atrial fibrillation D) Sinus bradycardia

C) Atrial fibrillation In patients with mitral valve stenosis, the pulse is weak and often irregular because of atrial fibrillation.

A student nurse is caring for a patient who has a diagnosis of acute pancreatitis and who is receiving parenteral nutrition. The student should prioritize which of the following assessments? A) Fluid output B) Oral intake C) Blood glucose levels D) BUN and creatinine levels

C) Blood glucose levels

A 33-year-old patient presents at the clinic with complaints of weakness, incoordination, dizziness, and loss of balance. The patient is hospitalized and diagnosed with MS. What sign or symptom, revealed during the initial assessment, is typical of MS? A) Diplopia, history of increased fatigue, and decreased or absent deep tendon reflexes B) Flexor spasm, clonus, and negative Babinskis reflex C) Blurred vision, intention tremor, and urinary hesitancy D) Hyperactive abdominal reflexes and history of unsteady gait and episodic paresthesia in both legs

C) Blurred vision, intention tremor, and urinary hesitancy

A nurse is caring for a patient with severe hemolytic jaundice. Laboratory tests show free bilirubin to be 24 mg/dL. For what complication is this patient at risk? A) Chronic jaundice B) Pigment stones in portal circulation C) Central nervous system damage D) Hepatomegaly

C) Central nervous system damage Prolonged jaundice, even if mild, predisposes to the formation of pigment stones in the gallbladder, and extremely severe jaundice (levels of free bilirubin exceeding 20 to 25 mg/dL) poses a risk for CNS damage.

A nurse is admitting a 45-year-old man to the medical unit who has a history of PAD. While providing his health history, the patient reveals that he smokes about two packs of cigarettes a day, has a history of alcohol abuse, and does not exercise. What would be the priority health education for this patient? A) The lack of exercise, which is the main cause of PAD. B) The likelihood that heavy alcohol intake is a significant risk factor for PAD. C) Cigarettes contain nicotine, which is a powerful vasoconstrictor and may cause or aggravate PAD. D) Alcohol suppresses the immune system, creates high glucose levels, and may cause PAD.

C) Cigarettes contain nicotine, which is a powerful vasoconstrictor and may cause or aggravate PAD.

A patient with portal hypertension has been admitted to the medical floor. The nurse should prioritize which of the following assessments related to the manifestations of this health problem? A) Assessment of blood pressure and assessment for headaches and visual changes B) Assessments for signs and symptoms of venous thromboembolism C) Daily weights and abdominal girth measurement D) Blood glucose monitoring q4h

C) Daily weights and abdominal girth measurement Obstruction to blood flow through the damaged liver results in increased blood pressure (portal hypertension) throughout the portal venous system. This can result in varices and ascites in the abdominal cavity. Assessments related to ascites are daily weights and abdominal girths.

A 30 year-old female patient has been diagnosed with Cushing syndrome. What psychosocial nursing diagnosis should the nurse most likely prioritize when planning the patients care? A) Decisional conflict related to treatment options B) Spiritual distress related to changes in cognitive function C) Disturbed body image related to changes in physical appearance D) Powerlessness related to disease progression

C) Disturbed body image related to changes in physical appearance

A nurse is addressing the prevention of esophageal cancer in response to a question posed by a participant in a health promotion workshop. What action has the greatest potential to prevent esophageal cancer? A) Promotion of a nutrient-dense, low-fat diet B) Annual screening endoscopy for patients over 50 with a family history of esophageal cancer C) Early diagnosis and treatment of gastroesophageal reflux disease D) Adequate fluid intake and avoidance of spicy foods

C) Early diagnosis and treatment of GERD Chronic esophageal irritation or GERD is among the most significant.

How should the nurse best position a patient who has leg ulcers that are venous in origin? A) Keep the patients legs flat and straight B) Keep the knees bent to 45-degree angle and supported with pillows C) Elevate the patients lower extremities D) Dangle the patients legs over the side of the bed

C) Elevate the patients lower extremities With venous insufficiency, dependent edema can be avoided by elevating the lower extremities. Dangling the patients legs and applying pillows may further compromise venous return.

The nurse is writing a plan of care for a patient with a cardiac dysrhythmia. What would be the most appropriate goal for the patient? A) Maintain a resting heart rate below 70 bpm. B) Maintain adequate control of chest pain. C) Maintain adequate cardiac output. D) Maintain normal cardiac structure.

C) Maintain adequate cardiac output.

A nurse is performing an admission assessment of a patient with a diagnosis of cirrhosis. What technique should the nurse use to palpate the patients liver? A) Place hand under the right lower abdominal quadrant and press down lightly with the other hand. B) Place the left hand over the abdomen and behind the left side at the 11th rib. C) Place hand under right lower rib cage and press down lightly with the other hand. D) Hold hand 90 degrees to right side of the abdomen and push down firmly.

C) Place hand under right lower rib cage and press down lightly with the other hand.

A patient who underwent surgery for esophageal cancer is admitted to the critical care unit following postanesthetic recovery. Which of the following should be included in the patients immediate postoperative plan of care? A) Teaching the patient to self-suction B) Performing chest physiotherapy to promote oxygenation C) Positioning the patient to prevent gastric reflux D) Providing a regular diet as tolerated

C) Positioning the patient to prevent gastric reflux

A patient with liver disease has developed jaundice; the nurse is collaborating with the patient to develop a nutritional plan. The nurse should prioritize which of the following in the patients plan? A) Increased potassium intake B) Fluid restriction to 2 L per day C) Reduction in sodium intake D) High-protein, low-fat diet

C) Reduction in sodium intake

A local public health nurse is informed that a cook in a local restaurant has been diagnosed with hepatitis A. What should the nurse advise individuals to obtain who ate at this restaurant and have never received the hepatitis A vaccine? A) The hepatitis A vaccine B) Albumin infusion C) The hepatitis A and B vaccines D) An immune globulin injection

D) An immune globulin injection

An older adult patient with type 2 diabetes is brought to the emergency department by his daughter. The patient is found to have a blood glucose level of 623 mg/dL. The patients daughter reports that the patient recently had a gastrointestinal virus and has been confused for the last 3 hours. The diagnosis of hyperglycemic hyperosmolar syndrome (HHS) is made. What nursing action would be a priority? A) Administration of antihypertensive medications B) Administering sodium bicarbonate intravenously C) Reversing acidosis by administering insulin D) Fluid and electrolyte replacement

D) Fluid and electrolyte replacement The overall approach to HHS includes fluid replacement, correction of electrolyte imbalances, and insulin administration.

A nurse is participating in the emergency care of a patient who has just developed variceal bleeding. What intervention should the nurse anticipate? A) Infusion of intravenous heparin B) IV administration of albumin C) STAT administration of vitamin K by the intramuscular route D) IV administration of octreotide (Sandostatin)

D) IV administration of octreotide (Sandostatin)

A patient with pheochromocytoma has been admitted for an adrenalectomy to be performed the following day. To prevent complications, the nurse should anticipate preoperative administration of which of the following? A) IV antibiotics B) Oral antihypertensives C) Parenteral nutrition D) IV corticosteroids

D) IV corticosteroids

A nurse is caring for a patient with liver failure and is performing an assessment in the knowledge of the patients increased risk of bleeding. The nurse recognizes that this risk is related to the patients inability to synthesize prothrombin in the liver. What factor most likely contributes to this loss of function? A) Alterations in glucose metabolism B) Retention of bile salts C) Inadequate production of albumin by hepatocytes D) Inability of the liver to use vitamin K

D) Inability of the liver to use vitamin K

A patient seeking care because of recurrent heartburn and regurgitation is subsequently diagnosed with a hiatal hernia. Which of the following should the nurse include in health education? A) Drinking beverages after your meal, rather than with your meal, may bring some relief. B) Its best to avoid dry foods, such as rice and chicken, because theyre harder to swallow. C) Many patients obtain relief by taking over-the-counter antacids 30 minutes before eating. D) Instead of eating three meals a day, try eating smaller amounts more often.

D) Instead of eating three meals a day, try eating smaller amounts more often.

The nurse is creating a plan of care for a patient who has a recent diagnosis of MS. Which of the following should the nurse include in the patients care plan? A) Encourage patient to void every hour. B) Order a low-residue diet. C) Provide total assistance with all ADLs. D) Instruct the patient on daily muscle stretching.

D) Instruct the patient on daily muscle stretching.

A medical nurse is aware of the need to screen specific patients for their risk of hyperglycemic hyperosmolar syndrome (HHS). In what patient population does hyperosmolar nonketotic syndrome most often occur? A) Patients who are obese and who have no known history of diabetes B) Patients with type 1 diabetes and poor dietary control C) Adolescents with type 2 diabetes and sporadic use of antihyperglycemics D) Middle-aged or older people with either type 2 diabetes or no known history of diabetes

D) Middle-aged or older people with either type 2 diabetes or no known history of diabetes HHS occurs most often in older people (50 to 70 years of age) who have no known history of diabetes or who have type 2 diabetes.

A nurse working in a long-term care facility is performing the admission assessment of a newly admitted, 85-year-old resident. During inspection of the residents feet, the nurse notes that she appears to have early evidence of gangrene on one of her great toes. The nurse knows that gangrene in the elderly is often the first sign of what? A) Chronic venous insufficiency B) Raynauds phenomenon C) VTE D) PAD

D) PAD In elderly people, symptoms of PAD may be more pronounced than in younger people. In elderly patients who are inactive, gangrene may be the first sign of disease.

A patient has been taking prednisone for several weeks after experiencing a hypersensitivity reaction. To prevent adrenal insufficiency, the nurse should ensure that the patient knows to do which of the following? A) Take the drug concurrent with levothyroxine (Synthroid). B) Take each dose of prednisone with a dose of calcium chloride. C) Gradually replace the prednisone with an OTC alternative. D) Slowly taper down the dose of prednisone, as ordered.

D) Slowly taper down the dose of prednisone, as ordered.

A 55-year-old man has been newly diagnosed with acute pancreatitis and admitted to the acute medical unit. How should the nurse most likely explain the pathophysiology of this patients health problem? A) Toxins have accumulated and inflamed your pancreas. B) Bacteria likely migrated from your intestines and became lodged in your pancreas. C) A virus that was likely already present in your body has begun to attack your pancreatic cells. D) The enzymes that your pancreas produces have damaged the pancreas itself.

D) The enzymes that your pancreas produces have damaged the pancreas itself.

Graduated compression stockings have been prescribed to treat a patients venous insufficiency. What education should the nurse prioritize when introducing this intervention to the patient? A) The need to take anticoagulants concurrent with using compression stockings B) The need to wear the stockings on a one day on, one day off schedule C) The importance of wearing the stockings around the clock to ensure maximum benefit D) The importance of ensuring the stockings are applied evenly with no pressure points

D) The importance of ensuring the stockings are applied evenly with no pressure points

A group of nurses are participating in orientation to a telemetry unit. What should the staff educator tell this class about ST segments? A) They are the part of an ECG that reflects systole. B) They are the part of an ECG used to calculate ventricular rate and rhythm. C) They are the part of an ECG that reflects the time from ventricular depolarization through repolarization. D) They are the part of an ECG that represents early ventricular repolarization.

D) They are the part of an ECG that represents early ventricular repolarization. ST segment is the part of an ECG that reflects the end of the QRS complex to the beginning of the T wave. The part of an ECG that reflects repolarization of the ventricles is the T wave. The part of an ECG used to calculate ventricular rate and rhythm is the RR interval. The part of an ECG that reflects the time from ventricular depolarization through repolarization is the QT interval.

A nurse is assessing a new patient who is diagnosed with PAD. The nurse cannot feel the pulse in the patients left foot. How should the nurse proceed with assessment? A) Have the primary care provider order a CT. B) Apply a tourniquet for 3 to 5 minutes and then reassess. C) Elevate the extremity and attempt to palpate the pulses. D) Use Doppler ultrasound to identify the pulses.

D) Use Doppler ultrasound to identify the pulses.

A nurse assesses clients at a community health fair. Which client is at greatest risk for the development of hepatitis B? a. A 20-year-old college student who has had several sexual partners b. A 46-year-old woman who takes acetaminophen daily for headaches c. A 63-year-old businessman who travels frequently across the country d. An 82-year-old woman who recently ate raw shellfish for dinner

a. A 20-year-old college student who has had several sexual partners Hepatitis B can be spread through sexual contact, needle sharing, needle sticks, blood transfusions, hemodialysis, acupuncture, and the maternal-fetal route. A person with multiple sexual partners has more opportunities to contract the infection.

A nurse cares for a client with hepatopulmonary syndrome who is experiencing dyspnea with oxygen saturations at 92%. The client states, I do not want to wear the oxygen because it causes my nose to bleed. Get out of my room and leave me alone! Which action should the nurse take? a. Instruct the client to sit in as upright a position as possible. b. Add humidity to the oxygen and encourage the client to wear it. c. Document the clients refusal, and call the health care provider. d. Contact the provider to request an extra dose of the clients diuretic.

a. Instruct the client to sit in as upright a position as possible.

A nurse reviews the chart and new prescriptions for a client with diabetic ketoacidosis: Vital Signs and Assessment Blood pressure: 90/62 mm Hg Pulse: 120 beats/min Respiratory rate: 28 breaths/min Urine output: 20 mL/hr via catheter Lab Results: Serum K 2.6 Meds: K 40 mEq IV bolus STAT Increase IV fluid to 100 mL/hr What action should the nurse take? a. Administer the potassium and then consult with the provider about the fluid order. b. Increase the intravenous rate and then consult with the provider about the potassium prescription. c. Administer the potassium first before increasing the infusion flow rate. d. Increase the intravenous flow rate before administering the potassium.

b. Increase the intravenous rate and then consult with the provider about the potassium prescription. The client is acutely ill and is severely dehydrated and hypokalemic. The client requires more IV fluids and potassium. However, potassium should not be infused unless the urine output is at least 30 mL/hr.

A nurse assesses a client with atrial fibrillation. Which manifestation should alert the nurse to the possibility of a serious complication from this condition? a. Sinus tachycardia b. Speech alterations c. Fatigue d. Dyspnea with activity

b. Speech alterations Clients with atrial fibrillation are at risk for embolic stroke.

A nurse assesses clients on the medical-surgical unit. Which client should the nurse identify as at high risk for pancreatic cancer? a. A 26-year-old with a body mass index of 21 b. A 33-year-old who frequently eats sushi c. A 48-year-old who often drinks wine d. A 66-year-old who smokes cigarettes

d. A 66-year-old who smokes cigarettes Risk factors for pancreatic cancer include obesity, older age, high intake of red meat, and cigarette smoking.

A nurse assesses a client who has diabetes mellitus. Which arterial blood gas values should the nurse identify as potential ketoacidosis in this client? a. pH 7.38, HCO3 22 mEq/L, PCO2 38 mm Hg, PO2 98 mm Hg b. pH 7.28, HCO3 18 mEq/L, PCO2 28 mm Hg, PO2 98 mm Hg c. pH 7.48, HCO3 28 mEq/L, PCO2 38 mm Hg, PO2 98 mm Hg d. pH 7.32, HCO3 22 mEq/L, PCO2 58 mm Hg, PO2 88 mm Hg

b. pH 7.28, HCO3 18 mEq/L, PCO2 28 mm Hg, PO2 98 mm Hg When the lungs can no longer offset acidosis, the pH decreases to below normal. A client who has diabetic ketoacidosis would present with arterial blood gas values that show primary metabolic acidosis with decreased bicarbonate levels and a compensatory respiratory alkalosis with decreased carbon dioxide levels.

A nurse assesses clients at a community health center. Which client is at highest risk for pancreatic cancer? a. A 32-year-old with hypothyroidism b. A 44-year-old with cholelithiasis c. A 50-year-old who has the BRCA2 gene mutation d. A 68-year-old who is of African-American ethnicity

c. A 50-year-old who has the BRCA2 gene mutation

A nurse assesses clients on the medical-surgical unit. Which client is at greatest risk for the development of carcinoma of the liver? a. A 22-year-old with a history of blunt liver trauma b. A 48-year-old with a history of diabetes mellitus c. A 66-year-old who has a history of cirrhosis d. An 82-year-old who has chronic malnutrition

c. A 66-year-old who has a history of cirrhosis The risk of contracting a primary carcinoma of the liver is higher in clients with cirrhosis from any cause.

A nurse prepares to assess the emotional state of a client with end-stage pancreatic cancer. Which action should the nurse take first? a. Bring the client to a quiet room for privacy. b. Pull up a chair and sit next to the clients bed. c. Determine whether the client feels like talking about his or her feelings. d. Review the health care providers notes about the prognosis for the client.

c. Determine whether the client feels like talking about his or her feelings.

A nurse assesses a client with Alzheimers disease who is recently admitted to the hospital. Which psychosocial assessment should the nurse complete? a. Assess religious and spiritual needs while in the hospital. b. Identify the clients ability to perform self-care activities. c. Evaluate the clients reaction to a change of environment. d. Ask the client about relationships with family members.

c. Evaluate the clients reaction to a change of environment.

A nurse assesses a client who is being treated for hyperglycemic-hyperosmolar state (HHS). Which clinical manifestation indicates to the nurse that the therapy needs to be adjusted? a. Serum potassium level has increased. b. Blood osmolarity has decreased. c. Glasgow Coma Scale score is unchanged. d. Urine remains negative for ketone bodies.

c. Glasgow Coma Scale score is unchanged. best indicator of therapy effectiveness for HHS

A nurse cares for a client with acute pancreatitis. The client states, I am hungry. How should the nurse reply? a. Is your stomach rumbling or do you have bowel sounds? b. I need to check your gag reflex before you can eat. c. Have you passed any flatus or moved your bowels? d. You will not be able to eat until the pain subsides.

c. Have you passed any flatus or moved your bowels? Paralytic ileus is a common complication of acute pancreatitis. The client should not eat until this has resolved.

After teaching a client who has plans to travel to a non-industrialized country, the nurse assesses the clients understanding regarding the prevention of viral hepatitis. Which statement made by the client indicates a need for additional teaching? a. I should drink bottled water during my travels. b. I will not eat off anothers plate or share utensils. c. I should eat plenty of fresh fruits and vegetables. d. I will wash my hands frequently and thoroughly.

c. I should eat plenty of fresh fruits and vegetables. The client should be advised to avoid fresh, raw fruits and vegetables because they can be contaminated by tap water.

A nurse cares for a client who has chronic cirrhosis from substance abuse. The client states, All of my family hates me. How should the nurse respond? a. You should make peace with your family. b. This is not unusual. My family hates me too. c. I will help you identify a support system. d. You must attend Alcoholics Anonymous.

c. I will help you identify a support system.

A nurse prepares to discharge a client with Alzheimers disease. Which statement should the nurse include in the discharge teaching for this clients caregiver? a. Allow the client to rest most of the day. b. Place a padded throw rug at the bedside. c. Install deadbolt locks on all outside doors. d. Provide a high-calorie and high-protein diet.

c. Install deadbolt locks on all outside doors.

A nurse is teaching the daughter of a client who has Alzheimers disease. The daughter asks, Will the medication my mother is taking improve her dementia? How should the nurse respond? a. It will allow your mother to live independently for several more years. b. It is used to halt the advancement of Alzheimers disease but will not cure it. c. It will not improve her dementia but can help control emotional responses. d. It is used to improve short-term memory but will not improve problem solving.

c. It will not improve her dementia but can help control emotional responses.

A nurse is teaching a client with chronic migraine headaches. Which statement related to complementary therapy should the nurse include in this clients teaching? a. Place a warm compress on your forehead at the onset of the headache. b. Wear dark sunglasses when you are in brightly lit spaces. c. Lie down in a darkened room when you experience a headache. d. Set your alarm to ensure you do not sleep longer than 6 hours at one time.

c. Lie down in a darkened room when you experience a headache.

A nurse assesses a client who has cholecystitis. Which clinical manifestation indicates that the condition is chronic rather than acute? a. Temperature of 100.1 F (37.8 C) b. Positive Murphys sign c. Light-colored stools d. Upper abdominal pain after eating

c. Light-colored stools Jaundice, clay-colored stools, and dark urine are more commonly seen with chronic cholecystitis.

A nurse cares for a client experiencing diabetic ketoacidosis who presents with Kussmaul respirations. Which action should the nurse take? a. Administration of oxygen via face mask b. Intravenous administration of 10% glucose c. Implementation of seizure precautions d. Administration of intravenous insulin

d. Administration of intravenous insulin The rapid, deep respiratory efforts of Kussmaul respirations are the bodys attempt to reduce the acids produced by using fat rather than glucose for fuel. Only the administration of insulin will reduce this type of respiration by assisting glucose to move into cells and to be used for fuel instead of fat.

A nurse cares for a client who is hemorrhaging from bleeding esophageal varices and has an esophagogastric tube. Which action should the nurse take first? a. Sedate the client to prevent tube dislodgement. b. Maintain balloon pressure at 15 and 20 mm Hg. c. Irrigate the gastric lumen with normal saline. d. Assess the client for airway patency.

d. Assess the client for airway patency.

A nurse assesses a client with a neurologic disorder. Which assessment finding should the nurse identify as a late manifestation of amyotrophic lateral sclerosis (ALS)? a. Dysarthria b. Dysphagia c. Muscle weakness d. Impairment of respiratory muscles

d. Impairment of respiratory muscles In ALS, progressive muscle atrophy occurs until a flaccid quadriplegia develops. Eventually, the respiratory muscles are involved, which leads to respiratory compromise.

After hiatal hernia repair surgery, a client is on IV pantoprazole (Protonix). The client asks the nurse why this medication is given since there is no history of ulcers. What response by the nurse is best? a. Bacteria can often cause ulcers. b. This operation often causes ulcers. c. The medication keeps your blood pH low. d. It prevents stress-related ulcers.

d. It prevents stress-related ulcers.

A nurse cares for a client who presents with an acute exacerbation of multiple sclerosis (MS). Which prescribed medication should the nurse prepare to administer? a. Baclofen (Lioresal) b. Interferon beta-1b (Betaseron) c. Dantrolene sodium (Dantrium) d. Methylprednisolone (Medrol)

d. Methylprednisolone (Medrol)

A nurse assesses a client who is prescribed an infusion of vasopressin (Pitressin) for bleeding esophageal varices. Which clinical manifestation should alert the nurse to a serious adverse effect? a. Nausea and vomiting b. Frontal headache c. Vertigo and syncope d. Mid-sternal chest pain

d. Mid-sternal chest pain Mid-sternal chest pain is indicative of acute angina or myocardial infarction, which can be precipitated by vasopressin.

A nurse is caring for a client with acute pericarditis who reports substernal precordial pain that radiates to the left side of the neck. Which nonpharmacologic comfort measure should the nurse implement? a. Apply an ice pack to the clients chest. b. Provide a neck rub, especially on the left side. c. Allow the client to lie in bed with the lights down. d. Sit the client up with a pillow to lean forward on.

d. Sit the client up with a pillow to lean forward on.

A client has returned to the nursing unit after an open Nissen fundoplication. The client has an indwelling urinary catheter, a nasogastric (NG) tube to low continuous suction, and two IVs. The nurse notes bright red blood in the NG tube. What action should the nurse take first? a. Document the findings in the chart. b. Notify the surgeon immediately. c. Reassess the drainage in 1 hour. d. Take a full set of vital signs.

d. Take a full set of vital signs.

A nurse cares for a client who has been diagnosed with the Huntington gene but has no symptoms. The client asks for options related to family planning. What is the nurses best response? a. Most clients with the Huntington gene do not pass on Huntington disease to their children. b. I understand that they can diagnose this disease in embryos. Therefore, you could select a healthy embryo from your fertilized eggs for implantation to avoid passing on Huntington disease. c. The need for family planning is limited because one of the hallmarks of Huntington disease is infertility. d. Tell me more specifically what information you need about family planning so that I can direct you to the right information or health care provider.

d. Tell me more specifically what information you need about family planning so that I can direct you to the right information or health care provider. The presence of the Huntington gene means that the trait will be passed on to all offspring of the affected person. Client may require genetic counselor or reproductive specialist

A nurse cares for a client with amyotrophic lateral sclerosis (ALS). The client states, I do not want to be placed on a mechanical ventilator. How should the nurse respond? a. You should discuss this with your family and health care provider. b. Why are you afraid of being placed on a breathing machine? c. Using the incentive spirometer each hour will delay the need for a ventilator. d. What would you like to be done if you begin to have difficulty breathing?

d. What would you like to be done if you begin to have difficulty breathing?

A client is being taught about drug therapy for Helicobacter pylori infection. What assessment by the nurse is most important? a. Alcohol intake of 1 to 2 drinks per week b. Family history of H. pylori infection c. Former smoker still using nicotine patches d. Willingness to adhere to drug therapy

d. Willingness to adhere to drug therapy


Conjuntos de estudio relacionados

BIO 102 - ASSESSMENT QUESTIONS (FINAL)

View Set

CWNA Chapter 3: Radio Frequency Components, Measurements, and Mathematics

View Set

Monopolistic Competition and Oligopoly

View Set

Observational Behavior- Ch 5 Foundations of Employee Motivation

View Set

Dr. Bailey Interpersonal Communication Final Exam

View Set

Drugs for Osteoarthritis/ Drugs used in Rheumatoid Arthritis (MDM)

View Set

Ch 19: Program Design and Technique for Speed and Agility Training

View Set

Principles of Management Exam 2 Ch.7-8

View Set